UWORLD (neuroanatomy)

Réussis tes devoirs et examens dès maintenant avec Quizwiz!

26 years man -Motorbike accident= maxillofacial injury -Difficulty chewing food (after surgery) -Jaw deviates to right when opening mouth -Foramen of nerve injured? a)Foramen lacerum b)Foramen ovale c)Foramen rotundum d)Foramen spinosum e)Jugular foramen

answer: foramen ovale -Sensory innervation to face and motor innervation to muscles of mastication= CN V3 -Muscles that close jaw (masseter, medial pterygoid, temporalis) and open jaw (lateral pterygoid) -Unilateral CN V3 injury= unopposed action of contralateral pterygoid muscles= deviation of mandible toward paralyzed side on opening the mouth -CN V3 exits through foramen ovale, which also contains lesser petrosal nerve, accessory meningeal artery and emissary veins option A= meningeal branch of ascending pharyngeal artery, emissary veins; internal carotid passes over it option C= CN V2 option D= middle meningeal artery and vein, and meningeal (recurrent) branch of CN V3 which supplies dura and contains sympathetic fibers option E= IX, X, XI, inferior petrosal and sigmoid sinuses; posterior meningeal artery

3 years girl -Worsening fever and lethargy -Recurrent upper respiratory illness; pulling ears -Eating poorly, vomiting -In emergency dep.= tonic-clonic seizures -CT= solitary 2-cm ring-enhancing lesion in left lateral temporal lobe -Pathogen gained access via? a) ethmoid air cells b) frontal sinus c) mastoid air cells d) maxillary sinus e) nose and upper lips f) peritonsillar space

-brain abscess= contained area of liquefactive necrosis with surrounding inflammatory capsule and marked vasogenic edema (ring-enhancing lesion) -Presents= fever, headache, focal neurological findings (e.g., seizure) -Most cases= bacterial (e.g., viridans streptococcus, staphylococcus aureus)= from underlying adjacent or distant infection -1) simple abscess= direct invasion of a contiguous infection; patient likely had otitis media that spread to mastoid air cells and then invaded temporal lobe= temporal lobe abscess (frontal lobe abscess= from ethmoid or frontal sinusitis= options A and B) -2) multiple abscesses= hematogenous dissemination of distant infection, most commonly the heart (e.g., endocarditis) or lungs (e.g., abscess, empyema) option D= spread to orbit and cause periorbital or orbital cellulitis and abscess option E= drain into the ophthalmic veins and can lead to cavernous sinus thrombosis option F= fever, sore throat, muffled voice; complications include airway obstruction, spread of infection to deep neck spaces or mediastinum

68 years man blockage of ACA (angiography) action impaired? a)Climbing stairs b)Gripping c)Speaking d)Swallowing food e)Whistling

answer: climbing stairs -Internal carotid arteries= labelled in blue; MCA (in green) and ACA (red arrow) -ACA: supplies medial region of ipsilateral hemisphere (frontal pole to parietooccipital sulcus) -Occlusion of ACA= contralateral leg and foot (sensory and motor) -Bilateral occlusion of ACA= behavioral symptoms (e.g., abulia) and urinary incontinence if frontal micturition center (e.g., medial frontal lobe/cingulate gyrus) is affected option B, C, D, E= MCA; also, hemineglect, anosognosia, conjugate gaze deviation toward side of stroke, contralateral homonymous hemianopia

23 years woman -Acute attack of optic neuritis -History: relapsing-remitting multiple sclerosis -MRI- demyelination involving the right optic nerve -Found on examination of both eyes? learn not answer (options are in pic)

answer: A -Unilateral optic neuritis= afferent pupillary defect due to demyelination of right optic nerve -Optic nerve= afferent signals to pretectal area (midbrain)= activates both Edinger-Westphal nuclei= send efferent (parasympathetic) fibers down oculomotor nerve= innervates pupillary sphincter muscle of each iris -Crossover= light entering one eye= both eyes constrict pupils; ipsilateral eye (direct); contralateral eye (consensual) -Afferent pupillary defect (optic neuritis, retinal detachment) have absent bilateral pupillary constriction when shining light into affected eye -Shining light into normal eye= constriction of both eyes option B and D= defect in efferent portion of pathway carried by CN III option C= no specific condition

62 years right handed man -Left leg weakness (resolved within 30 mins) -Transient vision loss in right eye -Atherosclerotic plaque in extracranial portion of supplying artery -Percutaneous stenting of lesion= vascular catheter is inserted into the right common femoral artery and gradually inserted to level of aortic arch -Pathway of catheter? a)Aorta --> brachiocephalic artery --> common carotid artery --> external carotid artery b)Aorta --> brachiocephalic artery --> common carotid artery --> internal carotid artery c)Aorta --> common carotid artery --> external carotid artery d)Aorta --> common carotid artery --> internal carotid artery e)Aorta --> subclavian artery --> vertebral artery

answer: Aorta --> brachiocephalic artery --> common carotid artery --> internal carotid artery -Transient ischemic attacks, which often occur due to emboli originating from atherosclerotic plaques -Internal carotid artery= blood to motor cortex (controls contralateral limbs) -Ophthalmic artery, gives rise to retinal artery, originates from ipsilateral internal carotid artery -Left leg weakness and vision loss in right eye suggests lesion in right internal carotid artery -Aorta --> brachiocephalic artery --> right subclavian artery and right common carotid --> external carotid (face and neck) and internal carotid (supplies brain) option A= supplies face and neck not brain option C and D= left side option E= supply upper spinal cord, brainstem, cerebellum and posterior part of brain, not motor cortex or eye

46 years man -Upper limb weakness and numbness -Chronic neck pain (motor vehicle accident 4 years ago) -absent biceps reflex -Spinal segments involved? a) C3-C4 b) C5-C6 c) C7-C8 d) C8-T1 e) T1-T2

answer: C5-C6 -Absent biceps reflex -Test: quickly tapping hammer against biceps brachii tendon as it passes through cubital fossa= active muscle stretch receptors that communicate via musculocutaneous nerve with lower motor neurons in the anterior horn of the C5-C6 spinal level= biceps muscle contraction and jerking of forearm -C5-C6: also control brachioradialis reflex -Stronger than normal reflex = UMN lesion; absent or decreased reflex= LMN lesion option C= triceps reflex

67 years woman -Visual field test: right nasal hemianopia -MRI/magnetic resonance angiography= right internal carotid artery aneurysm -Which part of visual pathway affected? (learn don't solve a) A b) B c) C d) D e) E f) F g) G h) H

answer: D -Humans have max. horizontal field of view= 200 degrees with both eyes= central 120 degrees (binocular vision, both eyes) and 2 side fields of about 40 degrees seen by only 1 eye (monoscopic vision) -Light from the nasal visual field strokes the temporal side of each retina and optic nerve fibers from temporal part of retina= travel laterally via optic chiasm to pass into ipsilateral optic tract -Lesions involving lateral aspects of optic chiasm can cause ipsilateral nasal hemianopia (aneurysm of internal carotid artery)

45 years woman -Symptoms prominent when playing tennis -Diminished sensation in lateral palm, thenar eminence, palmar aspect of first 3 and half digits of right hand -Injury of nerve that courses between? a)Biceps and coracobrachialis muscles b)Flexor carpi ulnaris and flexor digitorum profundus muscles c)Humeral and ulnar heads of the pronator teres muscle d)Olecranon and the medial epicondyle of the humerus e)Supinator muscles and the head of the radius

answer: Humeral and ulnar heads of the pronator teres muscle -Median nerve injury (when it passes between pronator teres (pronator teres syndrome)) -Exacerbated by repetitive, forceful pronation (e.g., tennis) -1) lateral cords 2) brachial groove: b/w biceps brachii and triceps brachii) 3) b/w 2 heads of pronator teres 4) b/w flexor digitorum superficialis and profundus 5) below flexor retinaculum (through carpal tunnel) -Injury= 1) impaired sensation of first 3 and half digits (palmar) 2) thenar eminence sensation impairment 3) wrist flexion/abduction 4) 2nd and 3rd digit flexion option A= musculocutaneous nerve; trauma, shoulder dislocation= sensory loss over lateral forearm and elbow flexion option B= ulnar nerve; sensory loss in dorsomedial hand, medial palm, and medial ½ digits; wrist adduction/flexion and fingers adduction/abduction, 4th and 5th digit flexion option D= cubital tunnel; ulnar nerve option E= deep branch of radial nerve; sensory loss in posterior forearm, dorsolateral hand, and dorsal thumb

48 years man -Burning pain in left thigh and leg (erythematous, tender, fluid-filled vesicles that gradually crust later appear there) region: lower anteromedial thigh and medial leg -Dermatome? a) L1-L2 b) L3-L4 c) L5-S1 d) S2-S3 e) no specific dermatome pattern

answer: L3-L4 -Single and grouped vesicles in various stages of ulceration and crusting= herpes zoster (i.e., shingles) -Following primary VZV infection (e.g., chickenpox)= virus migrates via sensory nerves to the dorsal spinal ganglia, where it lies dormant for years -Reactivation of virus typically occurs in a single (or multiple-adjacent) ganglion= hemorrhagic inflammation involving entire peripheral nerve associated with that ganglion and eruption of a painful, vesicular rash on skin supplied by that sensory nerve (i.e., dermatome) -Rash involves lower anteromedial thigh and medial leg= L3-L4 option A= L1= inguinal ligament region; L2= upper anteromedial thigh, doesn't extend to leg option C= L5= lateral thigh, anterolateral leg; dorsum, sole of foot; S1= posterior thigh and leg, and lateral portion of foot option D= S2= posterior thigh, and leg, perineum, penis; S3= perineum and penis

question 2 of 2 -Diagnosis: pure sensory stroke -Improved on treatment, returned home -5 years later= dies of large MI -Autopsy= two 5- to 6-mm cavities in the deep structures of her brain filled with clear fluid -process? a)Autoimmune demyelination b)Embolic carotid atherosclerosis c)Embolic mitral valve disease d)Lipohyalinosis with small vessel occlusion e)Malignant cell infiltration

answer: Lipohyalinosis with small vessel occlusion - lacunae= small cavitary lesions occlude small vessels (e.g., lenticulostriate arteries)= in chronic hypertension and diabetes affect basal ganglia, internal capsule, pons cerebellum result: lipohyalinosis + microatheromas --> eventually liquefactive necrosis option A= multiple sclerosis; appear grossly as pink patches in white matter tracts option B and C= bacterial endocarditis= cortical infarcts, small emboli can cause lacunar infarcts but less likely option E= space-occupying lesion rather than small cavities; center of large tumor may cavitate due to insufficient perfusion, with resultant necrosis and hemorrhage

66 years man -Transient, painless vision loss in left eye -Stenosis of left internal carotid artery (atherosclerosis) -Carotid endarterectomy= left glossopharyngeal nerve is transected -Findings? a)Deviation of the protruded tongue toward left b)Hoarseness due to left vocal cord dysfunction c)Impaired taste sensation from anterior two-thirds of the tongue d)Loss of general sensation in the tonsillar lining e)Reduced salivary secretion from the submandibular gland

answer: Loss of general sensation in the tonsillar lining -Glossopharyngeal nerve= exits via jugular foramen; functions: -1) somatic motor: stylopharyngeus muscle only (elevates larynx during swallowing) -2) parasympathetic: inferior salivatory nucleus --> CN IX --> otic ganglion --> travels along auriculotemporal nerve (CN V) --> parotid gland secretion -3) general sensory: tympanic membrane (inner surface; middle ear), eustachian tube, posterior third of tongue, tonsillar region, upper pharynx (afferent portion of gag reflex), carotid body, carotid sinus -4) special sensory (taste): posterior third of tongue option A= CN XII option B= CN X; particularly recurrent laryngeal nerve option C= facial nerve option E= from superior salivatory nucleus; carried by facial nerve

28 years man -Tinnitus and hearing loss -MRI= bilateral vestibular schwannomas -Genetic testing= NF type 2 -Additional imaging= small meningioma at C5 spinal level impinging upon medial portion of dorsal columns -Findings? a)Decreased biceps tendon reflexes b)Loss of crude touch sensation in the legs c)Loss of pain sensation in the hands d)Loss of position sensation in the fingers e)Loss of vibration sensation in the toes

answer: Loss of vibration sensation in the toes -Dorsal columns= vibration and proprioception, fine touch (2-point discrimination) -Medial fibers= lower spinal levels; lateral fibers= higher spinal levels -Gracile fasciculus (medially)= afferent info from spinal cord below T8 (lower half of body) to gracile nucleus in medulla -Cuneate fasciculus (laterally)= afferent info from spinal cord above T6 (upper half of body) to cuneate nucleus in medulla option A= radiculopathy (C5 or C6); or other conditions affecting LMNs option B= anterior spinothalamic tract option C= lateral spinothalamic tract option D= lateral portion of dorsal columns

12 years boy -Right hand clumsiness -He attempted to climb tree several hours and lost balance and fell from branch -Tried to break the fall by grabbing the tree branch by his right hand -He landed safely with no head injury or loss of consciousness -Exam: difficulty performing fine finger movements with right hand -Structure likely injured? a)Axillary nerve b)Long thoracic nerve c)Lower trunk of the brachial plexus d)Musculocutaneous nerve e)Radial nerve f)Suprascapular nerve

answer: Lower trunk of the brachial plexus -Sudden upward jerking of arm at shoulder can cause injury to the lower trunk of the brachial plexus -Lower trunk= C8-T1= contributes to ulnar and median nerves= together they innervate all of the intrinsic muscles of hand (e.g., lumbricals, interossei, thenar and hypothenar) -Injury to lower trunk= paralysis of all intrinsic hand muscles (Klumpke's palsy) -Weakness of lumbricals= impaired flexion at metacarpophalangeal joints and impaired extension at interphalangeal joints -Relative sparing of extrinsic flexors and extensors of hand= total claw hand deformity -Sensory loss can occur over medial aspect of hand/forearm -Involvement of T1 nerve root proximal to sympathetic trunk= Horner's syndrome (ipsilateral) option A= shoulder dislocations, proximal humerus fracture= weakness of deltoid and teres minor option B= axillary lymph node dissection= paralysis of serratus anterior= winging of scapula option D= weakened elbow flexion= biceps brachii and brachialis muscle option E= fracture of midshaft of humerus= posterior forearm and dorsolateral hand numbness with paralysis of wrist and finger extensors option F= supraspinatus and infraspinatus; problems with abduction and lateral rotation of arm, respectively

35 years man -Painful tongue sore; fever, myalgias and arthralgias -Works as driver; Unprotected sex with stranger (1 month ago) -Exam: rash over trunk and cervical lymphadenopathy -Ulcer located on the median sulcus of tongue, 2cm anterior to foramen cecum -Pain sensation of ulcer carried by? a)Chorda tympani b)Glossopharyngeal nerve c)Mandibular division of the trigeminal nerve d)Maxillary division of trigeminal nerve e)Vagus nerve

answer: Mandibular division of the trigeminal nerve -Acute HIV= rash, lymphadenopathy, fever, painful oral ulcers on tongue -Motor innervation: 1) CN XIII; with exception of palatoglossus (CN X) -General sensory: 1) anterior 2/3= CN V3 2) posterior 1/3= CN IX 3) posterior area of tongue root= CN X -Gustatory innervation (taste): 1) anterior 2/3= CN VII (chorda tympani) 2) posterior 1/3= CN IX 3) posterior area of tongue root and taste buds of larynx and upper esophagus= CN X

22 years man -Knife wound to neck; moderate distress, but alert -Exam: right-sided hemiplegia -Loss of proprioception and vibratory sensation below C8 dermatome (right side) -Left side= loss of pain and temperature sensation below T2 dermatome -MRI? a)Complete transection of the spinal cord b)Damage to the cervical dorsal roots on the right side c)Lesion of the dorsal columns on the right side d)Lesion of the medial longitudinal fasciculus on the right side e)Right-sided Hemisection of the spinal cord

answer: Right-sided Hemisection of the spinal cord -right-sided Hemisection at C8 level= Brown-Sequard syndrome, characterized by lesions in: -1) anterior horn (lower motor neuron): ipsilateral paralysis at level of lesion -2) lateral corticospinal tract (upper motor neuron): ipsilateral paralysis below level of lesion -3) dorsal column (gracile and cuneate): ipsilateral loss of vibration, proprioception and light touch (2-point discrimination) below level of lesion -4) spinothalamic tract causes contralateral loss of pain and temperature (lateral spinothalamic) and crude touch (anterior spinothalamic) sensation 1-2 levels below lesion -Spinal injury above T1 can also cause Horner syndrome (ptosis, miosis, anhidrosis) if there is damage to oculosympathetic pathway option A= bilateral loss of all sensation and voluntary movement below level of lesion option B= ipsilateral loss of sensation and areflexia from specific dermatomes affected option C= ipsilateral loss of vibratory, proprioceptive and light touch sensation below level of lesion option D= associated with internuclear ophthalmoplegia, commonly seen in multiple sclerosis; impaired horizontal eye movement and weak adduction of ipsilateral eye with simultaneous abduction nystagmus of contralateral eye

43 years woman -Acute lower back pain after dragging heavy box -Pain radiates down the right posterior thigh to the foot -Pain= shooting and 8/10 intensity -No bowel or bladder symptoms -Straight leg raise testing is positive on right -Right hip extension is weaker compared to left; knee jerk reflexes are 2+ and bilaterally symmetric; but right ankle jerk reflex is absent -Nerve root damaged? a) L2 b) L3 c) L4 d) L5 e) S1

answer: S1 -Presentation= sciatica (nonspecific term for low back pain that radiates down leg)= cause= compression of lumbosacral nerve roots (disc herniation or spinal foraminal stenosis) -Radicular pain is worse when leg is extended at knee and hip is passively flexed (straight leg test) -Sciatic nerve (L4-S3) and compression most often at L5 or S1 -S1 radiculopathy- 1) sensory loss (posterior thigh and calf to the lateral aspect of foot) 2) weakness on thigh extension (e.g., due to denervation of gluteus maximus), knee flexion (hamstrings) and foot plantar flexion (gastrocnemius) 3) absent ankle jerk reflex option A and B= pain down anterior thigh; weakness on hip flexion (iliopsoas); knee and ankle jerk reflexes remain intact option C= paresthesia down anterior thigh to medial lower leg and foot; weakness on knee extension (quadriceps) with diminished knee jerk reflex option D= paresthesia down lateral thigh and calf to dorsal foot; weakness of dorsiflexion and inversion (e.g., tibialis anterior); foot eversion (peroneus) and toe extension (extensor digitorum brevis)

64 years woman -Episodic urinary incontinence; overactive bladder -Symptoms persist after behavioral interventions (unable to tolerate medication) -Electrical stimulation (to help modulate neural control of urinary bladder muscle, urethral sphincter, and pelvic muscles) -Electrode placed near? a) nucleus accumbens b) subthalamic nucleus c) vagus nerve d) T6 nerve root e) L5 nerve root f) S3 nerve root

answer: S3 nerve root -Urge incontinence (overactive bladder)= neural stimulation augments neural signals to detrusor muscle and urethral sphincter (nerves derived from S2-S4 roots) - somatic innervation of pelvic floor muscles= decreases urethrovesical angle= compress urethra= stops urine flow -External urethral sphincter (EUS) under somatic control via pudendal nerve (S2-S4)= stimulation S2-S4= increase EUS tone= improve urinary continence -Pelvic splanchnic nerves (S2-S4) provide parasympathetic input to bladder and urethral sphincter (worsen incontinence) BUT also involved in bladder stretch-contraction reflex (neural stimulation= inhibits this reflex, improving continence) option A= mediates reward and pleasure; stimulation= treat addiction option B= stimulation= treat rigidity of Parkinson disease option C= regulates GI system; stimulation= improve gastric motility option D= contributes to greater splanchnic nerve (T5-T10)= sympathetic innervation to foregut to slow peristalsis); sympathetic stimulation of bladder= inhibits detrusor= T10-L2 option E= somatic innervation to muscles of lower limb (e.g., gluteal muscles, tibilias muscles, extensor hallucis longus

53 years man -Severe lower back pain (2 weeks ago, without trauma; worsening) -Worse with back flexion and raising the legs; radiates into left leg -Pinprick in the perianal area doesn't cause rapid contraction of anal sphincter -Nerve root responsible? a) T12 b) L2 c) L4 d) L5 e) S1 f) S4

answer: S4 -Worsening back pain + positive straight leg test= subacute lumbosacral radiculopathy (cause: herniated disk; less commonly= neoplasm, fracture, epidural hematoma/abscess) -Perianal anesthesia with loss of anocutaneous reflex= S2-S4 -Spinal cord terminates as conus medullaris (T12-S4) at the L1-L2 vertebrae -Nerves below this point (L3-S4) exit inferiorly of their respective intervertebral foramina (referred to as: cauda equina) -Lesions at L1-L5 vertebral bodies (T12-S4 nerve roots) symptoms: 1) radicular low back pain (i.e., radiates down one or both legs) 2) saddle/perianal anesthesia (S2-S4) 3) bowel and bladder dysfunction 4) lower extremity weakness -Conus medullaris syndrome (L1-L2 vertebrae): bilateral, symmetric lower extremity weakness with UMN signs (e.g., spasticity, hyperreflexia) -Cauda equina syndrome (below L2): flaccid, asymmetrical lower extremity weakness option A= lower abdominal or groin pain option B and C= L2-L4 contribute to femoral nerve; mediate patellar reflex; loss of quadriceps muscle action and anterior thigh sensory loss option D= sensory loss over lateral thigh and anterior calf option E= supplies Achilles' reflex along with sensation to posterior leg and posterolateral calf

64 years woman -Occlusion of right posterior inferior cerebellar artery -Findings? a)down-and-out right eye with impaired extraocular movements b)Left-sided motor weakness, including lower facial region c)Loss of conjugate lateral gaze on both sides d)Sensory loss over the right side of the face and left side of the body e)Vision loss over the left half of the visual field in both eyes

answer: Sensory loss over the right side of the face and left side of the body -Occlusion of the right posterior inferior cerebellar artery (arises from vertebral artery) -Occlusion of PICA= lateral medullary (Wallenberg) syndrome -Symptoms: 1) vertigo and nystagmus (vestibular nuclei) 2) ataxia (inferior cerebellar peduncle) 3) loss of pain and temperature sensation in the ipsilateral face (spinal trigeminal nucleus) and contralateral body (spinothalamic tract) 4) bulbar weakness (e.g., dysphagia, dysarthria) (nucleus ambiguus) -Horner syndrome (miosis, ptosis, anhidrosis) may also occur in ipsilateral eye due to damage of descending sympathetic nervous system fibers option A= damage to CN III; unopposed action of superior oblique (CN IV) and lateral rectus (CN VI); + contralateral lower facial weakness and hemiplegia= medial midbrain infarction due to occlusion of posterior cerebral artery (i.e., Weber syndrome) option B= contralateral motor cortex or internal capsule infarction due to occlusion of MCA) option C= internuclear ophthalmoplegia; damage to MLF, which can occur with occlusion of pontine artery; bilateral loss of conjugate faze= multiple sclerosis option E= homonymous hemianopia with macular sparing (collateral supply from MCA) Is caused by occlusion of PCA; homonymous hemianopia without macular sparing= damage to optic radiation or tract= occlusion of MCA, anterior choroidal

15 years girl -Recurrent episodes of focal onset seizures evolving to bilateral tonic-clonic seizures (several months) -Severe headaches; MRI shown (in answer) -Which of the following histopathological findings most likely present in abnormal lesion? learn don't answer a)Dense perivascular aggregates of atypical lymphoid cells b)Infiltrating macrophages containing degraded myelin debris c)Intense inflammatory infiltrate around toxoplasma tachyzoites d)Necrosis with surrounding pseudopalisading malignant cells e)Tangle of large blood vessels with thickened walls

answer: Tangle of large blood vessels with thickened walls -Recurrent headaches and seizures + multiple flow voids in left temporoparietal lobe (MRI)= cerebral arteriovenous malformation (AVM) -AVMs are vascular malformations in which blood courses directly from arteries to veins, without passing via capillaries= these tangled vessels with turbulent flow can lead to aneurysm development, bleeding and damage to surrounding brain -Microscopy= abnormal vessels, including irregular arteries and large veins with thickened walls, and gliotic brain tissue -Frequently present in children and young adults with headaches and seizures as well as intracranial hemorrhage (e.g., intraparenchymal, subarachnoid hemorrhage); some are asymptomatic -A nest of abnormal vessels with "bag of worms" on angiography; MRI= prior hemorrhage and multiple dark flow voids option A= primary CNS lymphoma, typically diffuse large B-cell lymphoma; clinically= seizures, in older patients and immunocompromised; MRI= enhancing mass option B= in demyelinating plaques of multiple sclerosis; symptoms: variable; unilateral visual disturbance (e.g., optic neuritis) and focal sensory and motor deficits; MRI= periventricular white matter lesions option C= Toxoplasma gondii; necrotic debris with surrounding inflammation, tachyzoites and pseudocysts of bradyzoites; seizures and headaches are common; immunocompromised; imaging= ring enhancing lesions option D= glioblastoma; high grade infiltrating astrocytoma; presents with seizures, headache, neurologic deficits (adults); MRI= mass with central hemorrhagic necrosis, vasogenic edema, possible extension to contralateral hemisphere

35 years woman -Migraines (increasing in frequency and severity) -Imaging= small aneurysm arising from the segment of the right internal carotid artery within the cavernous sinus -If it expands, findings? a)Deviation of the tongue to the right when protruded b)Inability to contract the right orbicularis oculi muscle c)Loss of sensation from anterior two-thirds of the tongue d)Vision defect affect the left half of visual field e)Weakness of the right lateral rectus muscle

answer: Weakness of the right lateral rectus muscle -Cavernous carotid aneurysm (small= asymptomatic; enlarge= compress nearby structures, unilateral, throbbing headache and/or cranial nerve deficits) -The abducens nerve (CN VI) passes through cavernous sinus next to internal carotid artery= most likely damaged -Compression of CN VI= ipsilateral lateral rectus weakness= esotropia (inward eye deviation) and horizontal diplopia that is worse when looking toward side of lesion -Other nerves damaged: CN III, CN IV, and V1 and V2 branches (run through cavernous sinus) option A= (XII; innervates genioglossus; responsible for tongue protrusion; injury to right XII (most commonly due to carotid endarterectomy)= ipsilateral deviation of tongue option B= facial expression muscles; innervated by VII; damage to LMNs of facial nerve (e.g., parotid gland tumor, trauma)= complete paralysis of ipsilateral facial muscles option C= lingual nerve; V3 option D= homonymous hemianopia due to contralateral lesion in visual pathways posterior to optic chiasm (e.g., optic tract, radiations); cavernous carotid aneurysms can sometimes compress optic nerve or chiasm= ipsilateral monocular vision loss

32 years woman -Worsening headache and double vision -Exam: weakness of lateral rectus shown on CT -Which nerve involved? a) optic nerve b) oculomotor nerve c) trochlear nerve d) ophthalmic division of the trigeminal nerve e) abducens nerve

answer: abducens nerve -Lateral rectus originates at annular tendon and inserts onto the temporal surface of globe -1) oculomotor nerve (CN III)= superior rectus, medial rectus, inferior rectus, inferior oblique, levator palpebrae superioris; also parasympathetic innervation to ciliary muscle and iris sphincter= efferent pathway of pupillary reflex -2) trochlear nerve (CN IV) innervates superior oblique muscle= internally rotate and depress while adducted -3) abducens nerve (CN VI)= lateral rectus muscle= abducts eye -Acute sinusitis complication= cavernous sinus thrombosis due to spread of infection through valveless facial venous system= cause dysfunction of nerves in cavernous sinus (including extraocular muscles) option A= transmit visual info to brain; lesion= blindness option D= sensory info from forehead, side of nose, periorbital area and cornea

56 years man -Worsening lower extremity pain and weakness -Hematuria and intermittent burning on urination (past year) -Smoker (35 years); urine culture= negative; diagnosis: transitional cell carcinoma -CT= advanced bladder tumor compressing a nerve that passes through the obturator canal -Result? a)Abduction of the thigh b)Adduction of the thigh c)Extension of the leg d)Extension of the thigh e)Flexion of the thigh

answer: adduction of the thigh -Obturator nerve arises from lumbar plexus (L2-L4)= descends posteromedial to iliopsoas muscle and courses along lateral aspect of lesser pelvis before descending through obturator canal -Supplies: 1) obturator externus muscle and divides into anterior and posterior branches that supply the rest of the thigh adductor muscles (e.g., adductor longus, brevis, magnus) 2) anterior division= terminal cutaneous branch= distal medial thigh sensation -Causes: compression due to pelvic trauma, surgery or tumors (e.g., bladder cancer) option A= gluteus medius, minimus and tensor fascia lata= supplied by superior gluteal nerve= exits through greater sciatic foramen above piriformis option C= quadriceps femoris muscle group= femoral nerve; through femoral canal below midpoint of inguinal ligament option D= gluteus maximus= inferior gluteal nerve= exits via greater sciatic foramen below piriformis option E= psoas (lumbar plexus), iliacus (femoral nerve), sartorius muscles

49 years woman -3 months= worsening weakness of right hand; she has been tripping; twitching and cramping in her arms and legs -No sensory deficits; increased tone and fasciculations in the lower extremity muscles -Atrophy of the interosseus muscles of right hand and bilateral intrinsic muscles of feet -Physician taps on chin while mouth is slightly open= jaw jerks upward briskly -Cranial nerves mediating this reflex? a)afferent: glossopharyngeal; efferent: trigeminal b)Afferent: glossopharyngeal; efferent: vagus c)Afferent: trigeminal; efferent: facial d)Afferent: trigeminal; efferent: trigeminal e)Afferent: vagus; efferent: vagus

answer: afferent: trigeminal; efferent: trigeminal -Both upper (spasticity, increased tone) and lower (atrophy, fasciculations) motor neuron lesions= amyotrophic lateral sclerosis -Jaw jerk reflex= normally absent or very weak -Afferent: CN V3 à trigeminal mesencephalic nucleus à interneurons à trigeminal motor nucleus à efferent: CN V3 -Reflex above foramen magnum= bilateral lesions of UMNs above level of pons= abnormal reflex (e.g., ALS, multiple sclerosis, pseudobulbar palsy) option B= gag reflex; also, baroreceptor reflex= afferent (IX, X) and efferent (X) option C= CN V1; and CN VII= corneal reflex= causing blink option E= cough reflex

34 years man -Treatment for Hodgkin lymphoma -Severe vomiting, requires IV fluid -Which part responsible for his symptoms? a) thalamus b) superior and inferior colliculi c) ventral pons d) area postrema e) cerebellum

answer: area postrema -Vomiting reflex initiated by chemoreceptor trigger zone (CTZ, or area postrema) and nucleus tractus solitarius -Area postrema= dorsal surface of medulla at caudal end of 4th ventricle -Area doesn't have well developed BBB= allowing it to be exposed to chemicals -CTZ= 1) directly triggered by emetogenic substances (e.g., drugs, toxins) 2) indirectly by vagal afferents from GI tract -Chemotherapeutic agents (e.g., cyclophosphamide) cause damage to enterocytes, resulting in release of serotonin= stimulates 5-HT3 receptors on vagal afferent fibers= ascends to CTZ to trigger vomiting reflex -Chemotherapeutic drugs in CSF and blood can stimulate area postrema directly by causing local release of substance P, which activates neurokinin-1 (NK-1) receptors in brainstem to induce vomiting -Selective 5-HT3 receptor antagonists (e.g., ondansetron) and NK1 receptor antagonists (e.g., aprepitant, fosaprepitant) can be used against vomiting option B= superior and inferior colliculi are located in dorsal midbrain; involved in processing visual (superior) and auditory (inferior) info and initiate movement of head and eyes in response option C= ventral pons contains corticospinal tract (limb movement), corticobulbar tract (movements of head, face and neck) and abducens nucleus (horizontal eye movement option E= cerebellum= motor control (coordination, precise movements, posture) and cognitive functions (attention, language)

64 years smoker -Right shoulder pain (radiates to ipsilateral arm); weakness in right upper extremity (worsening) -Exam: awake, alert, and oriented; partial right-sided ptosis (fully intact extraocular movements); pupils asymmetric in dim light (both reactive to light)(symmetric in bright light) -Right upper extremity 3/5 strength and absent deep tendon reflexes -Patient's autonomic dysfunction lesion? a) autonomic ganglia b) brainstem c) cerebral cortex d) spinal cord e) subcortical gray matter f) vagus nerve

answer: autonomic ganglia -Smoking history, upper limb pain/weakness, ipsilateral ptosis and miosis= Pancoast tumor (X-ray) -Pancoast tumor (usually non-small cell lung cancers): arise near the superior sulcus (groove produced by subclavian artery) -Compression and invasion of the brachial plexus= ipsilateral shoulder pain, upper limb paresthesia, and areflexic arm weakness -Involvement of cervical sympathetic ganglia= Horner's syndrome -Horner syndrome= 1) partial ptosis (denervation of sympathetically controlled superior tarsal muscle of upper eyelid) 2) miosis (interruption of sympathetic fibers to dilator pupillae muscle; worst in dim light because higher sympathetic activity exacerbating the defect) 3) anhidrosis (loss of sympathetic innervation to facial sweat glands) -Sympathetic pathway (disruption= Horner)= hypothalamus (1st order neurons) --> brainstem --> C8-T2 (ciliospinal center of budge); intermediolateral column (2nd order neurons) --> anterior nerve roots (white communicating rami) --> superior cervical ganglion (3rd order neurons) --> postganglionic fibers (along carotid arteries) reaches target tissues in face and head

60 years man -Difficulties tasting food (bland) -Traumatic brain injury (head on motor vehicle collision; 2 months ago) -After accident= coma for several days; regained consciousness and began having severe headaches and impaired taste -No visual concerns, slurred speech, trouble swallowing, vertigo or extremity weakness or numbness -Cause? a)Anterior temporal lobe contusion b)Avulsion of olfactory nerve rootlets c)Ischemic stroke of the dorsolateral medulla d)Lingual nerve injury e)Sphenoid bone fracture

answer: avulsion of olfactory nerve rootlets -Traumatic brain injury= flavor perception depends on both smell and taste, with smell being more important -Anosmia (loss of smell)= describe difficulties with taste, even if taste sensation is intact -Perception of smell= 1) odorant molecules bind olfactory chemoreceptor cells in nasal mucosa 2) axons of chemoreceptor cells make up CN I rootlets 3) project through the cribriform plate of ethmoid bone 4) synapse on glomeruli of olfactory bulb 5) olfactory bulb projects 2nd order axons to primary olfactory cortex (medial temporal lobe via olfactory tract) -Head trauma can cause anosmia when acceleration-deceleration forces lead to avulsion of the olfactory nerve rootlets (as they traverse cribriform plate) option A= semantic memory; injury= impaired comprehension about meaning of words, pictures and objects option C= Wallenberg syndrome; vertigo/nystagmus, ipsilateral cerebellar signs (e.g., ataxia, dysmetria), loss of pain/temperature sensation in ipsilateral face and contralateral body, bulbar weakness (e.g., dysphagia), ipsilateral Horner syndrome option D= branch of CN V3; somatic sensation to anterior 2/3 of tongue; fibers of chorda tympani (CN VII) travel along lingual nerve (relay taste); if taste was affected then tongue numbness would occur too option E= damage to CN II, pituitary gland injury, and leakage of CSF into sphenoid sinus

25 years man -Right arm weakness (injured right shoulder in MVA; difficulty elevating arm since then) -Exam: diminished pinprick and temperature sensation over small area in the right upper lateral arm and prominent atrophy of right deltoid with 3/5 strength on right shoulder abduction -Injury of which nerve? a)Axillary b)Dorsal scapular c)Long thoracic d)Median e)Musculocutaneous f)Radial g)Thoracodorsal h)Ulnar

answer: axillary -Axillary nerve= originates from posterior cord of brachial plexus (carries fibers form C5 and C6) -Courses deep in axilla, below the shoulder joint, then runs through the quadrangular space (bounded by teres minor (superiorly), humerus (lateral); long head of triceps (medially), teres major (superior) -Innervates 1) deltoid (arm abduction) and teres minor 2) sensation over lateral shoulder -Most commonly due to shoulder trauma (e.g., anterior dislocation, proximal humeral fracture) option B= motor innervation to rhomboids (retract the scapula) and levator scapulae muscles (elevate the scapula) option C= axillary lymph node dissection; paralysis of serratus anterior= winging of scapula option D= supracondylar humerus fracture; palmar sensory loss over first 3 and half digits; thumb flexion/opposition; flexion of 2nd/3rd digits; wrist flexion/abduction option E= sensory loss over lateral forearm and weakened elbow flexion (biceps brachii and brachialis option F= at axilla (crutch palsy)= weakness of forearm, hand and finger extensor muscles (e.g., wrist drop, absent triceps reflex) with sensory loss over posterior arm, forearm and dorsolateral hand option G= latissimus dorsi; elbow extension, shoulder adduction and arm internal rotation option H= elbow injury; sensory loss over medial one and half digits of hand; weakness on wrist flexion/adduction, finger abduction/adduction, and flexion of 4th/5th digits

34 years woman -moderate hearing loss (several months) -Ringing in left ear (no recent infections or cold symptoms), no head trauma -Father= bilateral hearing loss (old age) -Exam: auditory canals (patent), tympanic membranes (gray with well-visualized light reflex); hearing diminished on left side -Left sided facial numbness, asymmetric smile, diminished corneal reflex (left eye) -Intracranial mass where? a)Above the sellar diaphragm b)Adjacent to the sagittal sinus c)Between the cerebellum and lateral pons d)In the dorsal midbrain e)Over the lateral cerebral convexity

answer: between the cerebellum and lateral pons -Unilateral hearing loss + facial numbness and weakness= vestibular schwannoma (most common cause of cerebellopontine angle tumor in adults) -most often arise from the vestibular potion of the vestibulocochlear nerve (CN VIII) -Spontaneous vestibular schwannomas (unilateral); bilateral= associated with NF type 2 -CN VII and CN V are in proximity to CN VIII= may be compressed -CN VIII= cochlear portion (sensorineural hearing loss and tinnitus), vestibular potion (unsteadiness and disequilibrium) -CN V= loss of ipsilateral facial sensation with interruption of corneal reflex (afferent) -CN VII= ipsilateral muscle facial muscle paralysis; corneal reflex (efferent) option A= craniopharyngiomas (arises from remnants of Rathke's pouch) or pituitary tumor; hypothalamic and pituitary dysfunction and changes in vision (bitemporal hemianopia) option B and E= meningioma; falcine/parasagittal region or over convexities of cerebral hemispheres; seizures, headaches, focal neurologic deficits option D= germ cell tumors arise from pineal gland in dorsal midbrain; obstructive hydrocephalus and Parinaud syndrome (e.g., upward gaze palsy)

54 years man -Difficulty speaking -Asked about onset of symptoms= answers "I.. Weak..morning" and becomes frustrated -Picked up pen, but trouble signing name -Mild weakness in right arm and face -Speech difficulties due to lesion in? a) frontal eye field b) broca area c) primary motor cortex d) primary somatosensory cortex e) wernicke area f) primary visual cortex

answer: broca area -Broca (motor, nonfluent) aphasia -Able to communicate meaningfully, but their speech is slow, and consists of nouns and verbs only (pauses as well) -Often have difficulty in writing and signing (aware of lesion) -Broca area is in the caudal part of inferior frontal gyrus of the dominant (usually left) hemisphere (Broadmann areas 44 and 45) -Patients may have associated right hemiparesis involving upper limb and face (close to primary motor cortex) option A= FEF; caudal end of middle frontal gyrus anterior to precentral sulcus (Broadmann areas 6 and 8); damage= eyes deviate to ipsilateral side option C= primary motor cortex, can cause slurred speech (paresis/paralysis of muscles of mouth, tongue and larynx) and contralateral hemiparesis; but no language deficit option D= primary somatosensory cortex; sensation loss in contralateral body option E= Wernicke area (Broadmann area 22); sensory aphasia; fluent aphasia (word salad); lack insight about their problem option F= visual cortex; cortical blindness or visual impairment that can make reading and writing difficult

38 years man -Behavioral changes= outbursts of anger, depressive mood, insomnia -Lack of focus and poor memory -Several years= involuntary, jerky, nonpurposeful movements of extremities and face (worsening; when awake, not when sleeping) -Mother= had similar abnormal body movements (in nursing home for dementia) -which area is affected? a) caudate nucleus b) anterior limb of internal capsule c) insula d) putamen e) globus pallidus

answer: caudate nucleus -Cognitive decline + chorea (jerky, nonpurposeful movements) + family history= Huntington disease -Autosomal dominant= excessive CAG trinucleotide repeats (gain of function mutation in the huntingtin gene)= causes loss of inhibitory (GABA) neurons in the caudate nucleus -Genetic analysis (confirmation); imagine studies= atrophy of caudate nuclei, causing enlargement of lateral ventricles -Head of caudate nucleus= inferolateral walls of the frontal horns of lateral ventricles -Caudate + putamen= striatum option B= internal capsule; anterior limb= thalamocortical fibers; posterior limb= corticospinal motor and somatic sensory fibers, visual and auditory fibers) option C= insula; role in limbic system (emotion), coordination of some autonomic functions option D= putamen atrophies in Huntington; but not as significant as caudate nucleus option E= atrophy of globus pallidus in late progression of Huntington when generalized cortical atrophy occurs

48 years man -Fever, progressive headache, double vision -Purulent nasal drainage and frontal headache (days) -Exam: ptosis, mydriasis, mild proptosis, loss of corneal reflex of right eye; unable to move right eye in any direction; decreased sensation of right upper face -Visual acuity: normal in both eyes -Structure involved? a) brainstem b) cavernous sinus c) frontal lobe of brain d) inferior orbital fissure e) jugular foramen f) optic chiasm

answer: cavernous sinus -Cavernous sinus thrombosis (direct spread of infection from medial third of face, sinuses (ethmoidal or sphenoidal) or teeth -or communicates into cavernous sinus retrograde via valveless facial venous system (via superior and inferior ophthalmic veins) -Pathogens: S. aureus (most common), streptococci, fungal organisms (rare; Mucor, Rhizopus) -Symptoms: 1) headache, fever, diplopia 2) CN III palsy (ptosis and mydriasis and eye movement (IV and VI too)) 3) ophthalmic and maxillary branches (CN V)= loss of upper facial sensation and afferent limb of corneal reflex 4) proptosis (eye protrusion) and chemosis (conjunctival swelling)= impaired venous drainage through ophthalmic veins option A= crossed sign; ipsilateral cranial nerve palsies, and contralateral hemiparesis; vertigo and ataxia option C= disinhibition; problems with planning/organization and personality changes option D= transmits CN V2, infraorbital vessels and branches from sphenopalatine ganglion option E= Vernet syndrome; dysfunction of IX, X, XI; dysphagia, hoarseness, loss of gag reflex on ipsilateral side, deviation of uvula toward normal side option F= bitemporal hemianopsia

28 years woman -Right hand tremors (several weeks); history: remitting-relapsing form of multiple sclerosis -Exam: no abnormal hand movement at rest -When asked to touch object on table= coarse tremor that gradually increases as the hand moves closer to its target -Dysfunction of? a) basal ganglia b) cerebellum c) motor cortex d) premotor cortex e) thalamus

answer: cerebellum -Large amplitude (coarse) tremor (worsens on approaching target)= cerebellar tremor -Pathologies affecting cerebellum: multiple sclerosis; stroke, fragile X syndrome, cerebellar degeneration -Cerebellar tremors: low frequency (<5Hz) and high amplitude (coarse); involve proximal and distal muscles -Called intention tremor (higher amplitude as approaching object) -Often accompanied by cerebellar signs (dysmetria, ataxia, impaired alternating movements/adiadochokinesia) option A= Parkinson disease; loss of dopaminergic neurons in basal ganglia (substantia nigra); resting tremor improves with movements; would be accompanied with rigidity (increased resistance to passive movements), bradykinesia (slowing of voluntary movements) and gait disturbances option C= spastic paralysis, clasp-knife rigidity (resistance during flexure); hyperreflexia; pathologic reflexes option D= abnormal posturing or repetitive movements option E= contralateral hemiplegia or contralateral hemisensory loss

9 years girl -Severe headache, lethargy and vomiting (weeks ago, progressively worsened) -CT= mass lesion, enlarged lateral and third ventricles, normal sized fourth ventricle -Obstruction site? a)Arachnoid villi b)Cerebral aqueduct c)Interventricular foramina of Monro d)Lateral foramina of Luschka e)Medial foramen of Megendie

answer: cerebral aqueduct -Choroid plexus (4, 1 in each ventricle)= composed of ependymal cell-covered outgrowths of pia matter capillaries (fenestrated, unlike other parts of CNS) -Ependymal layer= simple ciliated columnar epithelium with tight junctions in between cells (selective to material in blood that can enter CSF) -CSF à lateral ventricles à foramina of Monro à 3rd ventricle à cerebral aqueduct of Sylvius à 4th ventricle à 2 lateral foramina of Luschka and single foramen of Megendie à subarachnoid space à CSF returned to venous circulation via arachnoid villi (granulations) -patient= noncommunicating (obstructive) hydrocephalus -Causes: tumors, scarring, CNS malformations -Communicating hydrocephalus (children): global ventricular dilation and increased ICP (without blockage); impaired CSF absorption by arachnoid villi (inflammation, fibrosis, hemorrhage, infection)= option A; rarely due to increased CSF production (papillomas of choroid plexus) -Normal pressure hydrocephalus(adults): enlarged ventricles and normal ICP; impaired absorption of CSF is compensated by decreased CSF formation -Hydrocephalus ex vacuo: normal ICP and increased dilation of ventricles secondary to cortical atrophy= elderly with dementia Pseudotumor cerebri elevated ICP and classically occurs in young women who are overweight (etiology not well known option C= enlargement of lateral ventricle only option D and E= enlargement of all ventricles

69 years woman -Weakness of right foot -History: chronic hip pain (osteoarthritis); underwent left total hip arthroplasty while in right lateral decubitis position -Since surgery= difficulty moving right foot (unable to dorsiflex or evert the foot, or extend toes); loss of sensation over dorsum of right foot -Injury to? a) common fibular nerve b) deep fibular nerve c) femoral nerve d) lateral sural cutaneous nerve e) lumbosacral spinal nerve roots f) sciatic nerve g) tibial nerve

answer: common fibular nerve -Common fibular (peroneal) nerve= originates in popliteal fossa= courses around head and neck of fibula= divides into superficial and deep fibular nerves -Proximity to lateral neck of fibula= susceptible to compression from a leg cast or during prolonged time in the lateral decubitus position -Presents: 1) deep fibular (weakness of foot dorsiflexion and toe extension) 2) superficial fibular (weakness in foot eversion, sensory loss over dorsum of foot) -Exam: foot drop (loss of dorsiflexion) option C= weakness of hip flexion and knee extension; loss of sensation over anterior thigh option D= branch of common fibular nerve= sensation in posterolateral leg option F= most common during hip surgery; damage to tibial, fibular and sural nerves; loss of sensation over posterior log and weakness involving hamstrings and most muscles of lower legs option G= injured at knee by trauma or compression (baker cyst in popliteal fossa); weakness in foot plantar flexion and decreased sensation over sole of foot

6 years girl -Cerebral palsy, undergoes selective dorsal rhizotomy surgery -Born prematurely and had experience periventricular cerebral white matter injury -During procedure, dorsal rootlets of lumbosacral spinal nerves are selectively severed -Effect on lower extremities? a) decreased muscle mass b) decreased muscle tone c) increased tendon reflexes d) muscle fasciculations e) muscle paralysis

answer: decreased muscle tone -Cerebral palsy= heterogenous condition= permanent, nonprogressive motor dysfunction caused by damage to brain -Certain subtypes= significant spasticity (e.g., hypertonia, hyperreflexia) -Spastic cerebral palsy cause: periventricular white matter necrosis (in this patient)= leads to loss of descending inhibitory control from UMNs -Specifically, lack of CNS inhibition= hyperactive stretch reflex= stretch reflex is monosynaptic reflex mediated at level of spinal cord via: -1) muscle lengthening is sensed by muscle spindles -2) afferent signal of stretch is conveyed to the spinal cord via the dorsal nerve fibers -3) efferent signals are conveyed via ventral root of spinal cord -4) extrafusal fibers are stimulated, causing muscle contraction -Stretch reflex (function)= deep tendon reflexes (e.g., patellar, Achilles) and maintenance of muscle tone by constant muscular activity occurring even at rest -Hyperactive stretch reflex= increased muscle tone (e.g., spasticity) -Selective dorsal rhizotomy destroys afferent (sensory) arm of reflex arc, decreasing muscle tone without sacrificing motor innervation of muscle option A, D and E= leads to paralysis, muscle atrophy and fasciculations, ventral nerves are spared option C= decreased not increased

51 years woman -Right face felt funny; asymmetry of face when smiling, puffing out cheek or closing eyes tightly -Effacement of right nasolabial fold, mouth drawn to left side -Additional findings? a)Attenuated sense of touch on the right side of face b)Decreased tearing from the right eye c)Failure to elevated the palate when saying "ah" d)Inability to sweat on the right side of face e)Loss of taste sensation from the posterior 1/3 of the tongue

answer: decreased tearing from the right eye -Bell's palsy (idiopathic paresis of facial nerve) -Facial nerve function 1) motor output to facial muscles 2) parasympathetic innervation to the lacrimal, submandibular and sublingual salivary glands 3) taste from anterior 2/3 of tongue 4) somatic afferents from pinna and external auditory canal -Bell's palsy= 1) unilateral facial paralysis 2) impaired eye closure, eyebrow sagging, inability to smile and frown on affected side 3) disappearance of nasolabial fold, mouth drawn to non-affected side 4) decreased tearing, hyperacusis and/or loss of taste sensation over anterior 2/3 of tongue option A= CN V; CN V3 innervates muscles of mastication option C= a)palate elevation and gag reflex= CN IX (afferent) and CN X (efferent) option D= Horner's syndrome option E= CN IX

58 years man -Right upper extremity (4 weeks) -Frequently travels; difficulty carrying heavy bags into or out of overhead bins -Neck and shoulder pain (months) -Weakness of shoulder abduction, loss of biceps reflex on right side -Decreased sensation over right lateral arm and forearm -Pathologic process? a)Axonal loss in the corticospinal tracts b)Cystic expansion of the central canal c)Degenerative changes in the vertebral joints d)Microinjury and tear of the rotator cuff tendons e)Rupture of the long head of the biceps tendon

answer: degenerative changes in the vertebral joints -Sensory (e.g., lateral arm/forearm sensation loss) and motor deficits (impaired shoulder abduction, biceps reflex) are not isolated to the distribution of a single peripheral nerve but rather follow the C5 dermatomal and myotomal distribution, indicating cervical radiculopathy - depends on affected nerve root, but typically present with neck or arm pain associated with motor/sensory deficits -Occur due to compression of spinal nerve roots, which most commonly occur due to: -1) spinal spondylosis: aging leads to degeneration of vertebral bodies, discs and joints. Disc space decreases, and joint stress results in osteophyte (e.g., bone spur) formation, which slowly narrows the neural foramina and leads to subacute onset of symptoms (seen in this patient) -2) vertebral disc herniation: degeneration of ligamentous fibers can lead to a tear in the intervertebral disc annulus, resulting in herniation of nucleus pulposus with potential nerve root compression; occurs acutely (e.g., during weight-lifting) option A= occurs in amyotrophic lateral sclerosis, affects both UMNs and LMNs; it can result in weakness and atrophy, sensory disturbances would be atypical option B= syringomyelia; upper extremity weakness and sensory deficits; both upper extremities are typically affected (cape distribution) option D= weakness of shoulder abduction, sensory deficits are unexpected option E= shoulder pain that is aggravated by repetitive overhead activities; loss of biceps reflex if rupture is complete; wouldn't cause weakness of shoulder abduction (biceps flexes and supinates the forearm) and no sensory deficits

55 years man -Right shoulder pain (4 weeks); worsens when actively abducting the shoulder or when lying on the shoulder at night -Weakness with abduction and external rotation of right shoulder -MRI= partial rotator cuff tear -Before surgery= anesthesia b/w right anterior and middle scalene muscles to block C3-C7 nerve roots -Muscle affected by anesthesia? a) cricothyroid b) diaphragm c) intercostal d) platysma e) sternocleidomastoid

answer: diaphragm -Brachial plexus originates from C5-T1 (pass through scalene triangle in posterior neck) -Supplies all upper extremity except area of skin near axilla (innervated by intercostobrachial nerve/2nd intercostal nerve branch) and trapezius muscle (innervated by XI) -Interscalene nerve block= 1) shoulder and upper arm 2) phrenic nerve (C3-C5)= ipsilateral diaphragmatic paralysis option A= external branch of superior laryngeal nerve (X), helps tense vocal cords, allowing higher pitched speech option C= aid in expansion and contraction of chest cavity during breathing; intercostal nerves (ventral rami of thoracic spinal nerves option D= tenses skin of neck and lower face; cervical branch of VII option E= controlling neck flexion and head rotation; XI

65 years man -65 years man -Initial CT= small temporal lobe hemorrhage -Develops generalized tonic-clonic seizures -High BP, low pulse and respirations (with periods of apnea) -Repeat CT= acute hemorrhage expansion with brain herniation -Findings after 2nd CT? a) dilated pupil b) horner syndrome c) hyperreflexia d) nystagmus e) upward gaze palsy

answer: dilated pupil -Spontaneous intracerebral hemorrhage --> hemorrhage expansion --> Increased ICP --> headache, vomiting, impaired consciousness --> Cushing triad (e.g., hypertension, bradycardia, irregular respirations) --> brain herniation -Temporal lobe= transtentorial herniation of uncus --> compress CN III --> ipsilateral CN III palsy with a fixed dilated pupil due to preganglionic parasympathetic nerve fiber damage -Progression of hernia --> involve uncus bilaterally --> downward displacement of brainstem (i.e., central herniation) --> brainstem hemorrhages (i.e., from rupture of paramedian basilar artery branches; Duret hemorrhages) --> damage of medullary cardiac and respiratory centers --> death option C= a)uncal herniation can compress cerebral peduncles= contralateral hemiparesis (but flaccid paralysis with hyporeflexia (i.e., spinal shock)

65 years man -Suddenly had difficulty using fork and appeared confused -Patient is well oriented, answers questions appropriately and has no motor, sensory, or visual deficits -Left-right disorientation, unable to carry out simple calculations, read or write -Brain area involved? a)Bilateral occipital lobes b)Dominant parietal lobe c)Dorsolateral medulla d)Dorsolateral thalamus e)Medial medulla f)Midbrain at level of superior colliculus

answer: dominant parietal lobe -symptoms= ischemic stroke affecting angular gyrus of dominant parietal lobe (supplied by MCA) -Angular gyrus is part of parietal association cortex= integrates multisensory (e.g., visual, tactile, verbal) info to comprehend events and solve problems -Specifically, semantic processing, word reading and comprehension, and number processing -Damage= Gerstmann syndrome (agraphia, acalculia, finger agnosia, left-right disorientation) -Can occur as an isolated syndrome or lesions to angular gyrus (associated with alexia and aphasia) option A= cortical blindness; patients confabulate and deny blindness option C= PICA; Wallenberg syndrome; loss of pain/temperature over ipsilateral face and contralateral body (spinal trigeminal and spinothalamic tracts), ipsilateral bulbar muscle weakness (nucleus ambiguus), vertigo (vestibular nuclei), ipsilateral limb ataxia (inferior cerebellar peduncle) and Horner syndrome (descending sympathetic fibers) option D= contralateral hemisensory loss; weeks-months later= pain on side of deficit= thalamic pain syndrome option E= medial medullary syndrome= occlusion of anterior spinal artery; contralateral hemiparesis (lateral corticospinal tract), contralateral hemisensory loss (dorsal column medial lemniscal pathway) and ipsilateral tongue deviation (CN XII) option F= upward gaze palsy, absent pupillary light reflex, and impaired convergence (Parinaud syndrome)

23 years man complete transection of radial nerve action impaired? a)Abduction of the fingers b)Adduction of the thumb c)Extension of the wrist d)Flexion of the wrist e)Opposition of thumb

answer: extension of the wrist -Radial nerve= innervates extensor muscles of upper limb below the shoulder, and sensation to skin of posterior arm, forearm and dorsolateral hand -Injury= penetrating stab wound at humeral midshaft; can be injured within the axilla -Injury= weakness during wrist and finger extension (wrist drop) option A and B= ulnar option D= median or ulnar option E= median

34 years man -Difficulty hearing (3 weeks) -Difficult to tolerate everyday sounds; Ear pain; avoids public places -Injury to which nerve? a) accessory b) facial c) glossopharyngeal d) hypoglossal e) vagus

answer: facial -The middle ear cavity= 3 ossicles (malleus, incus, and stapes) and 2 skeletal muscles (tensor tympani and stapedius)= participate in transmission of sound from tympanic membrane to cochlea -Stapedius muscle innervated by stapedius nerve a branch of the facial nerve -Stapedius muscle= stabilize stapes --> injury= stapes oscillates more widely producing hyperacusis (increased sensitivity to everyday sounds; often withdraw socially) -Tensor tympani inserts into malleus= contracts the tympanic membrane medially, increasing its tension and dampening sound transmission= innervated by CN V3 -Ipsilateral hyperacusis is a common finding associated with Bell's palsy

36 years woman -Firm, nontender swelling of right cheek (4 months) -No fever, runny nose, sore throat or cough -Exam: fullness of preauricular space on right side -MRI= 2.2cm mass in right parotid gland (biopsy= neoplastic) -If untreated? a)Bitemporal hemianopsia b)Facial droop c)Facial numbness d)Hoarseness e)Horner syndrome f)Strabismus

answer: facial droop -Facial nerve (CN VII) (extracranial)= motor innervation of muscles of facial expression -Exits skull via stylomastoid foramen and courses within parotid gland (there divides into 5 terminal branches, temporal, zygomatic, buccal, mandibular and cervical) -Malignant parotid tumors= compress ipsilateral facial nerve= facial droop option A= craniopharyngioma and pituitary adenoma; compress optic chiasm option C= trigeminal nerve (3 branches)= sensation to face and motor innervation to mastication muscles option D= recurrent laryngeal nerve (branch of vagus nerve) loops below aortic arch on left and below subclavian artery on right= innervates laryngeal muscles; during thyroid surgery, this nerve damaged due to its proximity to inferior thyroid artery; unilateral nerve injury= hoarseness; bilateral injury= inspiratory stridor, respiratory distress due to vocal cord paralysis option E= a)ptosis, miosis, anhidrosis due to disruption of sympathetic innervation; lesions= lateral hypothalmus, hypothalamospinal tract, paravertebral sympathetic chain/stellate ganglion (e.g., Pancoast tumor) or internal carotid artery (e.g., carotid dissection) option F= disorders of extraocular muscles or nerves that innervate them (CN III, IV, or VI)

18 years woman -Fever, headache, lethargy, nausea, vomiting (2 days) -High pulse; diaphoretic, delirious, petechial rash on trunk and extremities -Signs of meningeal irritation; CT= no mass lesion -Lumbar puncture for CSF analysis; landmark to help locate site of needle insertion? a)Anterior superior iliac spine b)Costal margin c)Iliac crest d)Inferior angle of scapula e)Posterior superior iliac spine

answer: iliac crest -Fever + headache + delirium + petechial rash + meningismus= meningococcal meningitis -Diagnosis= confirmed by lumbar puncture after neuroimaging excludes mass lesion -Spinal cord until inferior border of L1 (adults; L2/L3 in infants) -Conus medullaris subsequently branches out into spinal nerves= cauda equina -Pia mater surrounds spinal cord, projects downwards to form filum terminale (filament connecting conus medullaris and posterior coccyx) -Dura extends below spinal cord termination= forms sac (with CSF, nerve roots and cauda equina)= ends at S2 and fuses with filum terminale to connect to the coccyx -Lumbar puncture in recumbent position (preferred) or sitting -Optimal location for needle insertion is L3/L4 or L4/L5 space (spinal cord termination site) -L4 lies on a line drawn between the highest points of the iliac crests, which can be visually identified and confirmed by palpation

21 years man -Head trauma (boxing); brief loss of consciousness (hit to right side of face) -Headache, muffled hearing (right ear) -Exam: alert and oriented; palpation of head= normal; cervical spine= normal -Otoscopic exam: hemotympanum (right ear) -Greatest risk of injury to? a) olfactory nerve b) oculomotor nerve c) trigeminal nerve d) facial nerve e) hypoglossal nerve

answer: facial nerve -Hearing loss + hemotympanum= temporal bone fracture -Temporal bone (many structures involved in hearing)= external auditory canal, tympanic membrane, ossicular chain, cochlea, vestibulocochlear nerve (CN VIII)= injury= hearing loss -facial nerve= brainstem --> internal auditory meatus (temporal bone) --> travels with CN VIII --> takes complicated route through temporal bone --> otic capsule (middle ear) --> mastoid portion --> exits via stylomastoid foramen -Facial nerve (risk of damage in temporal bone fracture) function --> 1) movement of face, including eye closure 2) taste from anterior 2/3 of tongue 3) parasympathetic innervation of submandibular and lacrimal glands 4) sound dampening (due to innervation of stapedius muscle) option A= olfactory nerve rootlets project through cribriform plate of ethmoid bone; acceleration-deceleration head injuries can lead to avulsion of olfactory nerve rootlets option B= exits via superior orbital fissure= risk with trauma to orbit or anterior face option C= divides into V1 (superior orbital fissure), V2 (foramen rotundum), V3 (foramen ovale); all foramina located in sphenoid bone; risk with trauma to orbit or anterior face option E= exits via hypoglossal canal of occipital bone; injured in fractures involving occipital bone; more commonly injured due to penetrating neck trauma

7 years boy -Injury to right knee; swollen right knee with abrasion of overlying skin -Distal motor and sensory innervation are intact, and popliteal and pedal pulses are normal -X-ray= nondisplaced patellar fracture -After put on cast; paresthesia and numbness of dorsum of right foot along with weakness in foot dorsiflexion and eversion -Site of nerve compression? a)Anterior compartment of the leg b)Fibular neck c)Lateral compartment of the leg d)Medial malleolus e)Popliteal fossa

answer: fibular neck -Sciatic nerve branches to common peroneal nerve and tibial nerve just proximal to popliteal fossa -Common fibular nerve courses around neck of fibula then divides into deep and superficial branches -Deep branch= foot and toe dorsiflexion (injury= foot drop), sensation to webspace between 1st and 2nd toes -Superficial branch= foot eversion and provides sensation to dorsum of foot and lateral shin -Injury= lateral blows to the knee, tightly applied plaster casts due to its superficial location at the neck of the fibula option A= compartment syndrome= fracture/casting= increased pressure in confined space can compromise blood flow; severe pain out of proportion; followed by weakness and sensory loss; anterior compartment syndrome= injury of deep fibular nerve option C= lateral compartment syndrome= injures superficial fibular nerve option D and E= courses popliteal fossa then tibial nerve innervates posterior muscles of leg; then passes through tarsal tunnel below the medial malleolus, provides sensation over sole and motor innervation to intrinsic foot muscles; popliteal injury= weakness on foot plantar flexion, foot inversion and to flexion; tarsal tunnel injury= sensory loss over sole, with intrinsic foot muscle weakness

4 days premature infant (30 weeks of gestation) -Hypotonic and less responsive -Lethargic infant with weak and high-pitched crying, prominent scalp veins, and tense fontanelles -Ultrasound= blood in lateral ventricles -Source of bleeding? a) bridging cortical veins b) germinal matrix c) meningeal arteries d) sagittal sinus e) vessels of the circle of Willis

answer: germinal matrix -Intraventricular hemorrhage (complication of prematurity) -IVH= almost always manifests first 5 days postnatally in infants before 32 weeks of gestation and/or birth weight <1500g -Can be clinically silent or present with altered level of consciousness, hypotonia, decreased spontaneous movements -If bleeding is more severe= bulging anterior fontanelle, hypotension, tonic-clonic seizures, irregular respirations and coma -Originates from germinal matrix (highly vascularized layer in the subventricular zone; source of neurons and glial cells during brain development)= lack support from glial fibers, making them more susceptible to hemorrhage -Between 24-32 weeks gestation, the germinal matrix starts to become less prominent, and its cellularity and vascularity decrease, leading to reduced risk of IVH in infants after 32 weeks option A= subdural hematoma option C= malignancy, hypercoagulable states, infection= thrombosis within the dural sinuses option D= epidural hematoma option E= rupture of saccular aneurysm= most common cause of subarachnoid hemorrhage; rare in infants; associated with blood in basal cisterns rather than lateral ventricles

65 years woman -Confused and kept asking repetitive questions (3 hours ago) -Exam: awake and alert but keeps asking, "what happened?" -She can state her date of birth and address correctly, recognizes family members, and is aware of current events -Patient recalls 0 of 3 objects after several minutes, can't retain the physician's name despite many repetitions -Speech is fluent without dysarthria or aphasia -Two hours later the patient spontaneously return to normal -Temporary dysfunction of? a) arcuate fasciculus b) hippocampus c) nucleus accumbens d) piriform cortex e) pontine reticular formation

answer: hippocampus -Sudden-onset confusion, memory loss, anterograde amnesia resolving within 2 hours is consistent with transient global amnesia -they also have varying degrees of retrograde amnesia (i.e., lack of recall of info prior to the episode) -Amnestic symptoms resolve fully (Except for events that occurred during the TGA episode itself) within 24 hours by definition -risk factors for TGA: older age, history of migraines -Etiology: unknown, but hippocampus dysfunction is implicated option A= association fiber tract connecting Broca and Wernicke area; lesion= conduction aphasia option C= mediates reward and pleasure; role in addiction option D= processing olfactory information option E= regulates arousal, attention, sleep and muscle tone; dysfunction= delirium, fluctuating levels of arousal

4 years boy -Headaches (worsening; month; most severe when lying down) -Nonbloody, nonbilious vomiting (2 weeks, morning) -Intermittent blurry vision (when looking up) -Exam: bilateral papilledema, inability to gaze upwards, bilateral eyelid retraction -Labs: sodium, GH, LH and FSH normal -Brain mass suspected and MRI ordered; most likely location of tumor? a) suprasellar area b) thalamus c) pineal gland d) pons e) cerebellum f) medulla

answer: pineal gland -Pineal gland mass -Classic manifestations: obstructive hydrocephalus from aqueductal stenosis (papilledema, headache, vomiting) and dorsal midbrain (Parinaud) syndrome due to compression of pretectal region of midbrain -Parinaud syndrome: limitation of upward gaze with downward gaze preference, bilateral eyelid retraction (e.g., Collier sign, sclera visible above the superior corneal limbus) and light-near dissociation (e.g., pupils react to accommodation but not light) -most common pineal mass is a germinoma, a midline malignant tumor that arises from embryogenic germ cells -Germinoma can arise in gonads or mediastinum or suprasellar region (less common site= option A)= due to their proximity to hypothalamus and pituitary, suprasellar germinomas do not lead to Parinaud syndrome, more likely to present with endocrinopathies (e.g., central precocious puberty, diabetes insipidus) option B= primary thalamic tumors are rare; more commonly damaged by infarction/hemorrhage= present with contralateral hemisensory loss, behavioral changes, and language deficits option D= pontine hemorrhage or tumor may cause locked in syndrome= spastic quadriplegia and paralysis of most cranial nerves with preserved consciousness and eye movements option E= cerebellar tumor= commonly medulloblastoma (in children)= gait and limb ataxia, intention tremor and nystagmus option F= a)lesions of medulla= lateral and medial medullary syndromes; lateral medullary (Wallenberg) syndrome leads to contralateral loss of pain and temperature sensations with ipsilateral paralysis of CNs V, IX, X and XI); medial medullar syndrome (contralateral spastic paralysis, contralateral sensory loss of vibration and proprioception, and ipsilateral flaccid paralysis of tongue (CN XII)

15 years boy -acute abdominal pain= acute appendicitis (3 weeks ago) -Appendectomy (no immediate complications) -Days later= burning pain at surgical scar radiating to suprapubic region but otherwise fine -Exam: healed surgical incision centered over McBurney point; loss of sensation over right suprapubic area -Cremasteric reflex is normal; nerve injured? a) femoral b) genitofemoral c) iliohypogastric d) ilioinguinal e) lateral femoral cutaneous f) obturator

answer: iliohypogastric -Iliohypogastric nerve (L1)= emerges from lateral border of upper psoas major, passes behind the kidney anterior to quadratus lumborum= provides motor function to anterolateral abdominal wall muscles= anterior branch emerges above the superficial inguinal ring to innervate suprapubic region= lateral branch descends over iliac crest to innervate gluteal region -Injury to anterior branch during appendectomy option A= arises from L2-L4; sensation to upper thigh and inner leg and innervates muscles that extend the knee option B= L1-L2; sensation to upper anterior thigh and motor function to parts of genitalia (e.g., cremasteric reflex in men, mons pubis in women) option D= L1; accompanies spermatic cord through superficial inguinal ring; sensation to upper and medial thigh and parts of external genitalia option E= L2-L3; sensation to skin on lateral thigh option F= L2-L4; skin of medial thigh; motor function to adductor muscles of thigh

24 years man -Burning and stabbing pain in left groin (underwent mesh repair of an inguinal hernia a year ago) -Light touching of the upper medial thigh or upper part of scrotum elicits burning pain -Cremasteric reflex is normal -Diagnosis= entrapment of nerve traversing superficial inguinal ring with spermatic cord -Blockade of which nerve= anesthetic relief? a) femoral b) genitofemoral c) iliohypogastric d) ilioinguinal e) obturator

answer: ilioinguinal -Inguinal hernia= protrusion of abdominal contents through abdominal wall above the inguinal ligament, via either the deep/internal inguinal ring and inguinal canal (indirect hernia) or the superficial inguinal ring (direct hernia) -Surgery= return herniated tissues to abdomen and close defect; synthetic mesh (e.g., polypropylene) is placed at site of defect= reduce tension on tissues and reduce risk of recurrence Postherniorrhaphy neuropathy= complication of hernia surgery (regional nerves may be injured during procedure (e.g., transection, cautery) or by postoperative inflammation and fibrosis (e.g., nerve entrapment) -Ilioinguinal nerve --> enters inguinal canal anterior to internal ring and follows spermatic cord (round ligament in women) to superficial ring; it gives sensory branches to --> 1) anterior scrotum (labia majora) 2) base of penis (mons pubis) 3) medial thigh option A= anterior thigh sensation; motor weakness of quadriceps option B= genital branch passes through inguinal canal; injury= pain at anterior scrotum; however, cremasteric reflex is mediated by the genitofemoral nerve option C= anterior branch of iliohypogastric nerve= pierces anterior abdominal musculature superior to superficial inguinal ring; sensory info= skin overlying the suprapubic/hypogastric region, not groin option E= innervates medial compartment of thigh; injury= weakness of hip adduction and decreased sensation of medial thigh; usually injured in pelvic (e.g., bladder) surgery

59 years man -Facial skin lesion (mole-like spot for several months, slowly enlarging) -Family history: skin cancer -Lesion: lateral to right nose above mustache; injection of anesthetic for biopsy where? a)Infraorbital foramen b)Mandibular foramen c)Mental foramen d)Stylomastoid foramen e)Superior orbital fissure

answer: infraorbital foramen -Lesion area supplied by maxillary branch of trigeminal nerve (CN V2) -Trigeminal nerve divides into 3 branches: -1) ophthalmic nerve (CN V1): exits the cranium through the superior orbital fissure (option E), passes superiorly along the orbit, then exits through the frontal bone via the supraorbital foramen -2) maxillary nerve (CN V2): leaves skull through foramen rotundum and then traverses pterygopalatine fossa to enter maxilla via the infraorbital canal; then exits facial bone through infraorbital foramen (infraorbital artery also exits via this foramen) -3) mandibular nerve (CN V3): exits cranium via foramen ovale; one branch (inferior alveolar nerve) enters mandibular foramen (choice B) and then exits at mental foramen (choice C) at anterior mandible -More general maxillary nerve block can be performed at pterygopalatine fossa (difficult, will block zygomatic and palatine nerves too) option D= facial nerve (CN VII) exits skull via stylomastoid foramen and provides motor innervation to muscles of facial expression

15 years girl -Severe left anterior leg pain (gymnastics) -Unable to bear weight on leg -Swelling of left knee with focal tenderness just above patella -Unable to raise let leg against gravity while in supine position -Imaging= complete tear of quadriceps tendon, but not fractures -Lidocaine injection where? a) adductor canal b) inguinal ligament c) ischial spine d) medial tibial condyle e) obturator canal

answer: inguinal ligament -Femoral nerve (L2-L4)= innervates quadriceps muscles (enabling extension at knee and flexion at hip) and provides sensation for arch of foot, shin and anteromedial thigh -Nerve emerges b/w psoas and iliacus muscles and passes under the inguinal ligament (lateral to femoral artery) into the thigh before branching in the femoral triangle into anterior and posterior division -Femoral nerve block= below inguinal ligament at lateral border of femoral artery= injecting here= anesthetizes the skin and muscles of anterior thigh, femur and knee; also, anesthetizes structures supplied by saphenous nerve to decrease sensation in the medial leg below the knee option A and D= saphenous nerve block option C= pudendal nerve block; anesthesia during vaginal deliveries and for minor surgeries of the vagina and perineum option E= obturator nerve block= anesthetize the adductor muscles of thigh and the skin of medial aspect of thigh

64 years woman -Lung cancer= SCC of right upper lobe (invaded pleura and 4th and 5th ribs) -Exam= right chest wall tenderness -Nerve block to control pain- injecting local anesthetic where? a) inferior cervical ganglion b) intercostal nerves c) phrenic nerve d) pulmonary plexus e) thoracic sympathetic trunk

answer: intercostal nerves -Chest wall pain due to cancer invading parietal pleura and ribs -Thoracic intercostal nerves (ventral rami of thoracic spinal nerves)= sensation from parietal pleura, ribs, and overlying skin -Intercostal vein, artery and nerve lie in the subcostal groove along the lower border of the rib and therefore provide a straightforward target for nerve block option A and E= sympathetic= no sensory info) option C= derived from C3-C5= motor innervation to diaphragm and pain fibers from diaphragmatic and mediastinal pleura (not costal pleura); often pain referred to base of neck and over shoulder= nerve block not recommended option D= branches of vagus nerve and sympathetic trunk; innervates visceral pleural= only detects stretch and not sensitive to pain

62 years woman -Had left mastectomy and axillary lymph node dissection (month ago; breast cancer) -Continued burning and aching in left upper arm -Exam: sensation diminished in medial upper arm, near axilla; shoulder range of motion is normal -Injury to? a) intercostobrachial b) medial pectoral c) suprascapular nerve d) thoracodorsal

answer: intercostobrachial -Axillary lymph node dissection= has persistent sensory dysfunction in her medial upper arm= intercostobrachial nerve injury -Intercostobrachial nerve= purely sensory nerve (originates from 2nd intercostal nerve's lateral cutaneous branch and traverses axilla to innervate skin of axilla and medial upper arm) -Most commonly injured in axillary lymph node dissection; other injuries are feared due to motor deficits like: -1) long thoracic nerve (serratus anterior)= scapular winging and weakness of arm abduction above horizontal line -2) thoracodorsal nerve (latissimus dorsi)= powerful adduction of arm, weakness while extending and medially rotating arm= option D -3) medial pectoral (pectoralis minor and major) and lateral pectoral (pectoralis major) nerves= weakness while adducting and medially rotating the arm, weakness flexing (clavicular head) or extending (sternocostal head) of the humerus= choice B option C= sensation to the shoulder joint (glenohumeral and acromioclavicular joints) and motor innervation of supraspinatus and infraspinatus muscles; from brachial plexus to suprascapular notch (prone to injury here= compression due to heavy backpack

62 years man -Right arm clumsiness -Exam: motor weakness involving right arm and leg -Speech is slurred, drooping of right lower face -Passive flexion of right arm= initial resistance followed by sudden release of tension as flexion is continued -Lesion location? a) caudate nucleus b) internal capsule c) insula d) putamen e) globus pallidus

answer: internal capsule -Patient has clasp-knife spasticity (initial resistance to passive flexion followed by sudden release of resistance)= seen with UMN lesions (lack of inhibition on spinal stretch reflex arc) -Passive arm flexion= extensor muscles are activated by the disinhibited stretch reflex= causing initial resistance that can be overcome with continued flexion -UMN lesions (any part of pyramidal motor system)= primary motor cortex, internal capsule, brainstem and spinal cord -Internal capsule stroke= sensory and motor deficits (most commonly pure motor stroke) affecting contralateral arm, leg and lower face (as well as clasp-knife spasticity, hyperreflexia, positive Babinski) -option A, D, E= lesions= chorea (random, jerky movements), tremor, bradykinesia, rigidity, changes in cognition and behavior option C= insula; role in consciousness and emotion (along with limbic system), ANS control, and awareness of visceral sensations

67 years woman -Hoarseness and difficulty speaking, loss of gag reflex on left side, uvula deviates to right side; hearing and vision= normal -Left shoulder is drooped, and strength is reduced during left shoulder shrug test -X-ray= right lower lobe lung mass and several osteolytic rib lesions; MRI (head)= multiple metastasis -Lesion location? a) cerebellopontine angle b) foramen magnum c) foramen ovale d) foramen rotundum e) hypoglossal canal f) jugular foramen

answer: jugular foramen -Jugular foramen (formed by petrous portion of temporal and occipital bones): IX, X, and XI -Results in jugular foramen (Vernet) syndrome: 1) loss of taste from posterior 1/3 of tongue (CN IX) 2) loss of gag reflex (IX, X) 3) dysphagia (IX, X) 4) dysphonia/hoarseness (X) 5) soft palate drop with deviation of uvula toward normal side (X) 6) sternocleidomastoid and trapezius muscle paresis (XI) -Option A= i.e., vestibular schwannoma= sensorineural hearing loss and tinnitus due to dysfunction of CN VIII

56 years man -Right leg pain and numbness (2 days ago, after 8-hour car trip) -Midway through car ride= numbness and burning pain over lateral aspect of right thigh -Lower extremity strength= 5/5 bilaterally -Right leg has large area of numbness over upper lateral thigh -Straight leg raise is negative; reflexes are intact -Compression of which nerve? a) ilioinguinal b) lateral femoral cutaneous c) obturator d) pelvic splanchnic e) sciatic

answer: lateral femoral cutaneous -Meralgia paresthesia= compression mononeuropathy of lateral femoral cutaneous nerve at inguinal ligament (tight clothing (belts), injury during surgery (hip arthroplasty); those with obesity and diabetes mellitus are at risk -LCFN (branch of lumbar plexus; L2-L3): lateral to psoas muscle and exits abdomen beneath inguinal ligament -MP= pain, paresthesia, numbness in lateral thigh (above knee); strength is normal option A= sensation to upper medial thigh and genital region; accompanies spermatic cord through superficial inguinal ring; injured during pelvic or hernia surgery= suprapubic or groin pain option C= anterior hip dislocation or pelvic surgery; sensation to medial thigh and weakness of adductor muscles option D= S2-S4; parasympathetic innervation to hindgut, bladder and urinary sphincters, regulate peristalsis, bladder emptying, and defecation; commonly involved in cauda equina syndrome (e.g., lumbar epidural metastasis) option E= L4-S3; compression at spine (e.g., herniated spine), piriformis muscle, greater sciatic foramen (hip surgery) or posterior thigh; symptoms; back and buttock pain that radiates down posterior thigh to foot, associated features= lower extremity weakness and positive straight leg raise test

20 years man -Right arm weakness and numbness -Competitive baseball pitcher (played more lately) -Difficulty using right arm, particularly when lifting objects -Diminished strength on right elbow flexion and absent biceps reflex -Sensory loss over? a) lateral forearm b) medial forearm c) posterior arm d) posterior forearm e) thenar eminence

answer: lateral forearm -Musculocutaneous nerve injury= 1) trauma (e.g., shoulder dislocation) 2) strenuous upper extremity activity (e.g., baseball pitching) -Derived from C5-C7 and lateral cord of brachial plexus -Innervates major forearm flexors (e.g., biceps brachii, brachialis) and coracobrachialis (flexes and adducts arm) -Lateral cutaneous branch= skin of lateral forearm option B= branch of medial cord; medial cutaneous nerve options C and D= posterior cutaneous branches of radial nerve option E= palmar cutaneous branch of median nerve

59 years man -Diplopia, drooping right eyelid (worsening) -Exam: dilated right pupil, nonreactive to both light and accommodation; vertical and horizontal diplopia -Asked to stare straight ahead= right eye is directed inferiorly and laterally with respect to left eye -MRI= aneurysm involving right posterior communicating artery -Muscle most likely to remain functionally intact? a)Inferior oblique b)Inferior rectus c)Lateral rectus d)Levator palpebrae e)Medial rectus f)Superior rectus

answer: lateral rectus -Patient has right oculomotor nerve (CN III) palsy secondary to compressive aneurysm -CN III lesions: ptosis, impair adduction, depression, and elevation of eye; diagonal (vertical and horizontal) diplopia -CN III lesion: eye rests in down and out position due to unopposed action of superior oblique and lateral rectus -Pupillary constriction and accommodation (Affected) as CN III carries parasympathetic fibers to ciliary muscle and iris sphincter -CN III palsy causes: intracranial tumors or aneurysms (most often posterior communicating artery), nerve ischemia, head trauma -Option A, B, D, E and F are innervated by CN III

55 years man -Progressive headaches (2 months) -Headaches: throbbing, associated with nausea, worsen when patient coughs or bears down during a bowel movement -History: episodic migraine without aura -MRI (with gadolinium= cystic mass in left lateral cerebellar hemisphere -Findings seen on physical exam? a)Left dysdiadochokinesia b)Left Horner syndrome c)Left oculomotor nerve palsy d)Photophobia e)Right hemiparesis f)Right hemisensory loss g)Right homonymous hemianopia h)Right lower facial droop

answer: left dysdiadochokinesia -Cystic neoplasm (likely hemangioblastoma) in the left cerebellar hemisphere= causing progressive headaches that worsen with Valsalva (maneuver increases ICP) -Cerebellar hemispheres= motor planning and coordination of the ipsilateral extremities (via descending lateral corticospinal tract, rubrospinal tract)= lesions= left dysdiadochokinesia (impaired rapid alternating movements); limb dysmetria (overshoot/undershoot during targeted movements) and intention tremor -Vermis= modulates axial/truncal posture and coordination via connections with the medial descending motor systems (anterior corticospinal, reticulospinal, vestibulospinal, and tectospinal tracts= lesions= truncal ataxia (wide based gait, unsteady gait) -Inferior vermis; and flocculonodular= lesion= vertigo and nystagmus option B= ptosis, miosis, anhidrosis; lesions affecting ipsilateral lateral hypothalmus or sympathetic tracts in brainstem (lateral medulla, carotid artery option C= mydriasis, ptosis, down and out deviation of eye; ipsilateral lesion at level of anterior midbrain or midbrain tegmentum option D= migraine with aura= less likely because it doesn't worsen with Valsalva option E, F, and H= cortical, subcortical, or upper brainstem lesions option G= optic tract, lateral geniculate body, optic radiations, primary visual cortex lesions

55 years man -History: poorly controlled hypertension and type 2 diabetes -Difficulty seeing; blurry vision (2 weeks ago); shadow across visual field of left eye (1 day ago) -No headaches, double vision, vertigo, light sensitivity or nausea -Exam: pupils equal and reactive to light -Area of reduced vision in left eye; flame-shaped hemorrhage in left temporal hemiretina -Transmission disrupted in? a)Left inferior colliculus b)Left lateral geniculate body c)Left medial geniculate body d)Medial optic chiasm e)Right inferior colliculus f)Right lateral geniculate body g)Right medial geniculate body

answer: left lateral geniculate body -Hypertensive hemorrhage in the left Temporal hemiretina -The left temporal hemiretina receives visual info from the nasal visual field of left eye -The visual signal is transmitted via the left optic nerve to the lateral aspect of optic chiasm. It then joins the visual signal from the right nasal hemiretina (receives visual info from temporal visual field of right eye) and travels via left optic tract to left lateral geniculate body in the thalamus = then travels via ipsilateral optic radiations to ipsilateral primary visual cortex option A, C, E, G= auditory pathway option D= transmit visual info from bilateral nasal hemiretinas option F= visual info from right temporal hemiretina and left nasal hemiretina

72 years man -Metastatic lung cancer; headache, nausea, vomiting -Squamous cell lung cancer with metastasis to right frontal lobe (in palliative care) -High BP, somnolent but wakes up to voice and follows instructions -CT (head)= hemorrhage within tumor, surrounding edema, right cingulate gyrus herniating beneath falx cerebri -Findings? a)Dilated and fixed right pupil b)Left lower extremity weakness c)Leftward conjugate eye deviation d)Loss of gag and cough reflexes e)Nonfluent./agrammatic speech

answer: left lower extremity weakness -Brain herniation= when acute increased ICP causes portion of brain to protrude through opening in the dural folds or foramen magnum -This patient= subfalcine herniation -Unlike with other herniations= some consciousness is usually preserved (no compression of brainstem centers (reticular activating system) -Initial presentation= headache, increased ICP (nausea, vomiting, lethargy) -Late= ipsilateral ACA compression= downstream ischemia and contralateral leg weakness option A= uncal herniation due to compression of ipsilateral CN III option C= classic finding in MCA stroke; due to damage to the frontal eye field; eyes directed ipsilateral to lesion option D= tonsillar herniation; compression of brainstem option E= expressive motor aphasia; injury to Broca area; stroke involving the superior division of the dominant MCA

65 years man -Acute-onset, right sided weakness and slurred speech -Severe headache, nausea -History: poorly controlled hypertension and chronic tobacco use -High BP and pulse -Exam: lethargic with right hemiparesis and lower facial weakness, right hemisensory loss, and dysarthria -Non-contrast CT shown -Vessels most likely affected? learn don't answer a) cortical branches b) inferior division of middle cerebral artery c) lenticulostriate arteries d) pontine arteries e) posterior cerebral artery f) superior division of middle cerebral artery

answer: lenticulostriate arteries -CT= hyperdense mass; consistent with acute putaminal hemorrhage (i.e., affecting basal ganglia)= almost always affect the adjacent internal capsule, leading to dysarthria, contralateral hemiparesis, and contralateral hemisensory loss due to disruption of corticobulbar, corticospinal and somatosensory fibers -As hemorrhage expands, it leads to increased ICP (headache, nausea/vomiting, altered mental status) -Hypertensive vasculopathy involving small, penetrating branches of the major cerebral arteries is the most common cause of spontaneous deep intracerebral hemorrhage -Chronic hypertension= Charcot-Bouchard aneurysms= rupture and bleed within deep brain structures= most frequent locations (basal ganglia, putamen, cerebellar nuclei, and pons); basal ganglia are supplied by the lenticulostriate arteries (deep, small vessel branches off the middle cerebral arteries) option A= affected in lobar hemorrhages (e.g., occipital) as opposed to deep hypertensive hemorrhages; spontaneous lobar hemorrhages occur in elderly due to amyloid angiopathy option B, E, F= middle (inferior and superior divisions) and posterior cerebral arteries are large, proximal vessels arising from circle of willis; associated with ischemic stroke due to thrombotic/embolic vessel occlusion or subarachnoid hemorrhage after saccular aneurysm rupture option D= small branches of basilar artery that can rupture in setting of poorly controlled chronic hypertension; pontine hemorrhages are close to midline and affect both sides of pons; present: coma (disruption of reticular activating system), locked in syndrome, and pinpoint pupils (descending sympathetic tract

26 years woman -Discomfort in right posterior mandible -Partially erupted right mandibular third molar tooth -Radiograph= impaction of molar tooth -During surgery= inferior alveolar nerve injured -Findings? a)Loss of general sensation in the anterior tongue b)Loss of sensation in the posterior tongue c)Loss of sensation in the lower lip d)Loss of taste sensation in anterior tongue e)Loss of taste sensation in the posterior tongue f)Weakness of the masseter muscle

answer: loss of sensation in the lower lip -Inferior alveolar nerve= branches from mandibular nerve (V3)= enters mandibular foramen to travel within mandibular canal (vulnerable during dental procedures) -From mandibular canal= small branches to provide sensory innervation to each of the lower teeth -Also gives off mental nerve= exits via mental foramen= sensation to chin and lower lip (skin and mucosal surfaces; unilateral) option A= lingual nerve; branch of CN V3; can be injured in procedures involving submandibular gland duct options B and E= CN IX option D= chorda tympani; branch of CN VII; travels with lingual nerve option F= masseteric branch of CN V3

43 years woman right handed MRI= tumor parasagital, medial portion of cerebral cortex findings? a)Bitemporal visual field defects b)Hand weakness and spasticity c)Lower limb sensory loss and hemineglect d)Tinnitus and unilateral hearing loss e)Unilateral ptosis and gaze palsy

answer: lower limb sensory loss and hemineglect -well-circumscribed parasagittal tumor= meningioma -Benign, slow growing (adults); arise near dural reflections (e.g., falx cerebri, tentorium cerebelli) -Parasagittal location= compress medial portion of cerebral cortex= affects lower extremities -Affecting postcentral gyrus (primary somatosensory cortex)= sensory loss in contralateral lower limb -Involvement of adjacent parietal association cortex (non-dominant hemisphere)= contralateral hemineglect (region involved in visuospatial processing) option A= compression of optic chiasm; pituitary adenoma or craniopharyngiomacompression of optic chiasm; pituitary adenoma or craniopharyngioma option B= meningiomas over lateral convexity; compressing lateral motor cortex option D= vestibular schwannoma; located at cerebellopontine angle option E= ptosis and down and out gaze; oculomotor nerve palsy, may occur with nerve damage due to compression (berry aneurysm, uncal herniation) or microvascular ischemia (e.g., diabetes mellitus)

62 years man -Memory impairment and frequent falls -Forgetful, didn't know location of various items in house (lived there 20 years) -Gait is unsteady; father is an alcoholic; father says he is fine -Exam: bilateral lateral gaze palsy and nystagmus -Memory testing: 0/3 objects after 5 mins -Lesion in? a) pons b) mammillary bodies c) fornix d) splenium of corpus callosum e) pineal gland f) cerebellum

answer: mammillary bodies -Alcohol abuse + oculomotor dysfunction (nystagmus, ophthalmoplegia), ataxia, and encephalopathy (confusion, anterograde amnesia)= classic triad of Wernicke encephalopathy, complication of thiamine (vitamin B1) deficiency -Thiamine= cofactor for many enzymes involved in glucose metabolism= deficiency= inability to use glucose for energy= neuronal damage and necrosis -Areas of high metabolic demand (susceptible)= mammillary bodies (focal hemorrhage or atrophy on imaging) -Other sites: thalamus, periaqueductal gray matter, locus coeruleus -Infusion of dextrose solutions (without thiamine supplementation)= worsen WE -Chronic effects of thiamine deficiency= Korsakoff syndrome= anterograde and retrograde amnesia, apathy, lack of insight, confabulation option A= demyelination of the pons (central pontine myelinolysis) can occur with overly rapid correction of chronic hyponatremia= locked-in syndrome, no verbal communication option C= fornix= bundles originating from hippocampal subiculum; projects to mamillary bodies; rarely can be involved in WE option D= splenium of corpus callosum; lesions (demyelinating or ischemic) results in alexia without agraphia option E= pineal gland= synthesized melatonin; circadian rhythm option F= alcoholic cerebellar degeneration also causes ataxia but doesn't cause confusion or ophthalmoplegia

26 years woman -Pain and muffled hearing in left ear (weeks), sensation of jaw clicking when chewing food -Episodic headaches, left-sided facial pain (several years; worse when moving jaw) -Grind's teeth while asleep; exam: unable to fully open mouth and pain with passive movement of jaw -Disorder affecting pterygoid muscle contractility suspected -Ear symptoms, which nerve? a) cervical spinal nerves b) facial nerve c) mandibular nerve d) maxillary nerve e) vestibulocochlear nerve

answer: mandibular nerve -Facial pain, headaches, jaw dysfunction, ear pain/muffled hearing= temporomandibular disorder -Etiology (multifactorial)= includes temporomandibular joint derangement (e.g., jaw misalignment, trauma) and hypersensitivity of the mandibular nerve -CN V3= sensation to TMJ, floor of mouth, anterior tongue, and lower parts of face; innervates muscles of mastication (e.g., medial and lateral pterygoid, masseter, temporalis), tensor veli palatini, and tensor tympani in middle ear (dampens loud sounds) option A= most people with TMD have cervical spinal dysfunction and neck point due to muscular functional connections between jaw and cervical regions; wouldn't cause ear pain; but rather neck, shoulder and arm pain/weakness option B= sensory innervation to external auditory canal/concha and motor innervation to stapedius muscle (dampens high amplitude sounds) option D= sensory innervation to cheek, nares, upper lip, and teeth; involvement of maxillary nerve in trigeminal neuralgia= facial pain option E= auditory and balance info from inner ear; damaged in vestibular schwannoma= hearing loss

23 years woman -Unconsciousness after motor vehicle accident (not wearing seatbelt) -High BP, low BP and respiration -Linear skull fracture at junction of frontal, parietal, temporal and sphenoid bones= CT -Branch of which of the following arteries most likely severed? a) facial b) maxillary c) middle cerebral d) occipital e) ophthalmic f) sphenopalatine

answer: maxillary -Fracture of pterion (bone is thin in this region)= risk of lacerating middle meningeal artery= epidural hematoma -Epidural hematomas require prompt treatment= they are under systemic arterial pressure= can expand rapidly, leading to elevated ICP (can cause Cushing reflex), brain herniation (e.g., uncal herniation with oculomotor nerve palsy) and death -Middle meningeal artery is a branch of the maxillary artery (one of terminal branches of external carotid artery)= enters skull via foramen spinosum= supplies dura mater and periosteum option A= branch of external carotid artery, courses over mandible anterior to insertion of the masseter muscle to supply the oral, nasal and buccal regions option C= branch of internal carotid artery; supplies parietal and temporal lobes; injury= subarachnoid or intracerebral hemorrhage option D= opposite of facial nerve from external carotid; supplies posterior scalp and sternocleidomastoid option E= first branch of internal carotid artery; supplies eye and orbital contents as well as eyelids, forehead and nose option F= branch of 3rd part of maxillary artery; supplies nasal mucosa; anastomoses with branches of ophthalmic and facial arteries within the anterior part of nasal septum (Kiesselbach's plexus (frequent site of nosebleeds)

3 years boy -Found underwater in bathtub (left alone for 5 mins) -Exam: doesn't respond to painful stimuli, pulseless, and not breathing -Cardiac rhythm= asystole and returns after several minutes of resuscitation -Endotracheal intubation= stimulation of posterior pharyngeal wall with laryngoscope elicits no soft palate movements or gagging -Dysfunction in? a) prefrontal cortex b) motor cortex c) midbrain d) medulla oblongata e) cervical spinal cord

answer: medulla oblongata -Absent gag reflex= mediated by 1) sensory (afferent) signals are sent via the CN IX 2) somatic sensory fibers of CN IX enter the brainstem at medulla; descend along spinal trigeminal tract, and synapse in the caudal spinal nucleus 3) information is then sent to bilateral nuclei ambiguus (in medulla) 4) bilateral motor (efferent) signals are sent via CN X= causing elevation of soft palate and contraction of pharynx -Dysfunction of medulla (progression= brain death) option A= inappropriate behavior, impaired judgement, poor problem-solving skills option C= pupillary reflex assessed in patients with suspected brain death= indicates dysfunction of the midbrain; mediated by afferent (CN II) and efferent (CN III) to mediate constriction of pupils in response to light option E= spinal reflexes; may be present even with brain death

65 years man -Cardiac arrest -comatose with fixed and dilated pupils -No direct or consensual pupillary response to light -MRI= diffuse loss of gray-white matter differentiation with sulcal effacement -Damage to is responsible for pupillary findings? a) medulla b) midbrain c) occipital lobe d) parietal lobe e) pons f) temporal lobe g) thalamus

answer: midbrain -MRI findings= anoxic brain injury due to cardiac arrest -Pupil findings= poor prognosis and indicates anoxic damage to the level of upper midbrain -Normal pupillary reflex= 1) retina and optic nerve and tract transmit light stimulus to midbrain at level of superior colliculus 2) received by pretectal nucleus 3) relayed to bilateral Edinger-Westphal nuclei 4) nuclei project preganglionic parasympathetic fibers through oculomotor nerve (CN III) to ciliary ganglion 4) projects postganglionic fibers that innervate the sphincter pupillae muscle (constricts pupil) -when light is shone in one eye= both ipsilateral (direct response) & contralateral (consensual response) pupils constrict option A= contains CN IX (afferent), CN X (efferent)= gag reflex option C= unilateral primary visual cortex damage= contralateral homonymous hemianopia; bilateral damage= cortical blindness option D= process and interpret visual, auditory and motor signals from other brain areas; damage= difficulties with spatial and visual perception; nondominant lesions= hemineglect and constructional apraxia (motor planning affected); dominant= Gerstmann syndrome (e.g., right-left confusion, agraphia, acalculia) option E= horizontal gaze center= mediates oculocephalic (doll's eye) reflex; also, CN V (afferent), CN VII (efferent)= corneal reflex; bilateral pons injury= pinpoint pupils due to damage of the descending sympathetic fibers option F= affects memory (hippocampus), speech (Wernicke area), vision (Meyer's loop), hearing (primary auditory cortex); bilateral damage to amygdala= Kluver-Bucy syndrome (hyperphagia, hyperorality, hypersexuality) option G= sensory relay station; VPL= sensation form body; VPM= facial sensation and taste; LGN= vision; MGN= hearing

54 years man -Unable to effectively communicate; speaks clearly and with conviction but his sentences are incomprehensible -He doesn't understand doctor's questions, does not follow oral or written instructions, and cannot repeat simple phrases -Artery occluded? a)Anterior cerebral artery b)Anterior inferior cerebellar artery c)Middle cerebral artery d)Posterior cerebral artery e)Posterior inferior cerebellar artery

answer: middle cerebral artery -Fluent aphasia, form of receptive aphasia (fluent speech but meaningless); can't understand verbal or written language; repetition is impaired (patients unaware of their deficit) -Lesion in Wernicke area (within auditory association cortex within posterior portion of superior temporal gyrus in dominant lobe); inferior branches of MCA supply it

75 years man -Unresponsive at home -History: poorly controlled hypertension -High BP and pulse; extensor posturing and pinpoint pupils -CT (head)= acute pontine hemorrhage with associate mass effect -Dies several hours later; autopsy= disruption of all pigmented neurons in the posterior rostral pons at lateral floor of 4th ventricle -Neurons produce? a)Dopamine b)Dynorphin c)Epinephrine d)Histamine e)Norepinephrine f)Orexin g)Oxytocin h)Vasopressin

answer: norepinephrine -Locus coeruleus= paired pigmented brainstem nucleus located in the posterior rostral pons near the lateral floor of 4th ventricle -Principle site for norepinephrine synthesis in the brain= involved in control of mood, arousal (reticular activating system), sleep-wake states, cognition, and autonomic function (e.g., blood pressure control) -Abnormal activation of locus coeruleus= pathogenesis of anxiety disorders -Bilateral pontine hemorrhage= coma due to disruption of reticular activating system= other features: total paralysis with extensor posturing due to corticospinal and corticobulbar tract injury and pinpoint pupils secondary to descending sympathetic tract damage option A= ventral tegmental area and substantia nigra pars compacta located in the midbrain; mesolimbic and mesocortical pathways (regulate cognition and behavior), nigrostriatal pathway (regulates coordination of voluntary movements), tuberoinfundibular pathway (inhibits prolactin secretion) option B= opioid peptide=modules pain response; produced in periaqueductal area, rostral ventral medulla and dorsal horn of spinal cord option C= adrenal medulla option D and F= posterior hypothalamus; role in arousal and wakefulness

19 years man -4-cm laceration over flexor surface of right proximal forearm -Damaged nerve: courses between flexor digitorum superficialis and profundus muscles -Impaired? a)Adduction of the third and fourth digits b)Adduction of the thumb c)Extension of the metacarpophalangeal joints d)Extension of the wrist e)Opposition of the thumb f)Supination of the forearm

answer: opposition of the thumb -origin: spinal nerves C5-T1 (via medial and lateral cords of brachial plexus) -Course: 1) with brachial artery b/w biceps brachii and brachialis muscles 2) b/w heads of pronator teres 3) b/w flexor digitorum superficialis and profundus 4) through carpal tunnel below flexor retinaculum -Most common injury: carpal tunnel syndrome -Other causes of injury: trauma to upper forearm, supracondylar fracture, entrapment between heads of pronator teres -Results: 1) pain and numbness in the 3 and ½ lateral digits 2) decreased sensation over thenar eminence and weakness of flexion of wrist and second and third digits (i.e., preacher or benediction hand) 3) weakness of thumb flexion and opposition option A= palmar interossei= deep branch of ulnar nerve; courses between flexor digitorum profundus and flexor carpi ulnaris option B= adductor pollicis muscle= deep branch of ulnar nerve; courses between the flexor digitorum profundus and flexor carpi ulnaris option C= extensor digitorum= deep branch of radial nerve (posterior interosseous nerve)= runs in posterior forearm between deep and superficial extensor muscles option D= extensor carpi ulnaris and extensor carpi radialis longus and brevis= radial nerve= courses between the medial and lateral heads of the triceps brachii in the upper arm before splitting into deep and superficial branches option F= supinator= deep branch of radial nerve= same as C

32 years man -Episodes of paroxysmal supraventricular tachycardia -Focal atrial tachycardia originating in crista terminalis of right atrium -Radiofrequency ablation is planned to destroy ectopic focus -Nerve in close proximity to treatment site, at risk of injury? a)Accessory nerve b)Long thoracic nerve c)Phrenic nerve d)Recurrent laryngeal nerve e)Sympathetic trunk

answer: phrenic nerve -PSVTs= arrhythmias that occur intermittently and have abrupt onset and offset -Arise from regions of abnormal electrical activity that occur in the AV node, accessory pathways, or atria -Crista terminalis= located in right atrium, common site of origination of PSVT -Right phrenic nerve near right atrium= risk of injury= on X-ray injury is recognized by elevation of right hemidiaphragm option A= spinal root; run from upper spinal cord through foramen magnum, then join cranial roots= combined roots then exit via jugular foramen= innervates sternocleidomastoid and trapezius (risk of injury in cervical lymph node dissection option B= serratus anterior; breast surgery or chest-tube placement; winged scapula option D= branch of CN X; courses under aortic arch on left and brachiocephalic trunk on right, to provide motor, sensory and parasympathetic innervation to upper esophagus, trachea and portions of larynx; at risk during thyroid surgery option E= parallels spinal column; injury= Horner syndrome, and during spinal surgery

56 years woman -Sudden onset headache and partial loss of vision loss -History: atrial fibrillation -Visual defect: right homonymous hemianopia with macular sparing -Occlusion of? a) anterior cerebral b) cerebral retinal c) middle cerebral d) posterior cerebral e) posterior inferior cerebellar

answer: posterior cerebral -Consistent with occlusion of PCA, most likely from an embolic event (atrial fibrillation) -PCA supplies occipital lobe (striate cortex/primary visual cortex); receives info from contralateral visual field from ipsilateral lateral genicular nucleus via optic radiation -Cuneate gyrus (of striate cortex)= info from upper retina (lower visual field) -Lingual gyrus (of striate cortex)= info from lower retina (upper visual field) -PCA occlusion= contralateral homonymous hemianopia with macular sparing (receives collaterals from MCA) option A= contralateral weakness of lower extremity option B= loss of vision in affected eye option C= contralateral motor and sensory deficits (more pronounced in upper extremity) and homonymous hemianopia with macular involvement option E= lateral medullary (Wallenberg) syndrome; contralateral loss of pain and temperature sensation, ipsilateral deficits of CN V, VIII, IX, X and XI and Horner's syndrome

70 years woman -Left sided visual loss (morning) -Exam: left homonymous hemianopia with macular sparing -CT (head)= area of focal parenchymal hypoattenuation -Cerebral artery compromised? a)Anterior cerebral artery b)Anterior choroidal artery c)Artery of Percheron d)Basilar artery e)Middle cerebral artery f)Posterior cerebral artery

answer: posterior cerebral artery -Embolic stroke from underlying atrial fibrillation -CT= shows PCA infarct -PCA branches off basilar artery at level of pontomesencephalic junction and supplies CN III and IV and other structures in midbrain -Also supplies thalamus (contralateral paresthesia and numbness), medial temporal lobe, splenium of corpus callosum, parahippocampal gyrus, fusiform gyrus (right fusiform gyrus injury, at occipitotemporal junction= prosopagnosia) and occipital lobe -Contralateral hemianopia with macular sparing (MCA collaterals for macula); visual agnosia and dyslexia can occur option A= inferior frontal, medial frontal and superior medial parietal lobes, anterior 4/5 of corpus callosum, olfactory bulb and tract, anterior portions of basal ganglia, internal capsule option B= branch of internal carotid; posterior limb of internal capsule, optic tract, lateral genicular body, choroid plexus, uncus, hippocampus and amygdala option C= branches off of either the right or left PCA; supplies bilateral thalami, and dorsal midbrain; rare normal variant option D= most of brainstem and bilateral cerebellar hemispheres option E= lateral convexity of frontal, parietal and temporal lobes; also supplies deep subcortical structures like internal capsule and basal ganglia

55 years man -Right arm tingling and numbness (several hours) -No headache, muscle weakness, slurred speech nor difficulty ambulating -History: hypertension, paroxysmal atrial fibrillation, ischemic stroke without residual symptoms -Anticoagulation discontinued due to GI hemorrhage -High BP, pulse regular -Exam: right arm paresthesia which spreads to entire right side of body and he develops right sided convulsions, followed by bilateral tonic-clonic seizure -Structure of left cerebral hemisphere that symptoms originated from? a) frontal eye field b) broca area c) primary motor cortex d) primary somatosensory cortex e) wernicke area f) primary visual cortex

answer: primary somatosensory cortex -Atrial fibrillation + anticoagulation stopped= recurrent embolic stroke complicated by seizure -Stoke is one of most common causes of seizure, which occurs due to release of excitotoxic neurotransmitters in ischemic brain injury -Initial manifestations of seizure (vary based on brain region first affected= patient's initial right arm numbness and paresthesia was likely caused by focal onset seizure originating in the left somatosensory cortex (postcentral gyrus) -Then developed right sided convulsions due to spread of seizure along the lateral brain convexity to the primary motor strip (precentral gyrus)= option C -Shortly after, the seizure activity spread to both cerebral hemispheres= tonic-clonic convulsions option A= frontal eye field; controls horizontal eye movement; activation of left FEF during seizure would cause eyes to deviate horizontally to right; after seizure, the patient's eyes may temporarily deviate to the left due to postictal neuroinhibition option B= Broca area; dominant hemisphere; responsible for speech; seizure here= uncontrolled vocalization or expressive aphasia option E= Wernicke area; dominant hemisphere; language comprehension; seizure here= fluent aphasia with impaired comprehension or auditory auras option F= primary visual cortex; visual info processing from contralateral homonymous visual fields; seizure here= visual hallucinations affecting right homonymous visual fields

34 years man -Physician ask him to stand with his feet close together, arms to sides, and eyes closed -Maneuver tests for which abnormality? a)Cortical sensory integration b)Gait c)Motor coordination d)Muscle strength e)Proprioception

answer: proprioception -Romberg test is a test of proprioception used to distinguish sensory from cerebellar ataxia -Legs together, arms to sides, eyes open= unable to maintain stance= ataxia (cerebellar or sensory) present -Ask to close eyes and maintain posture (visual inputs gone, and relies on proprioceptive receptors)= failure to maintain posture= positive Romberg sign= sensory ataxia -Causes of sensory ataxia: defects in posterior column or peripheral nerves (tabes dorsalis/syphilis; or vitamin B12 deficiency) -Patients that show little or no change with eyes closed (negative Romberg sign)= cerebellar ataxia due to impaired motor coordination (evaluated with rapid alternating movements, finger to nose testing, heel to shin testing)= choice C option A= parietal association cortex; nondominant hemisphere= hemineglect, anosognosia (lack of awareness of illness) and constructional apraxia (inability to draw objects); patients asked to draw a clock or bisect a horizontal line option B= asked to walk casually, in tandem, on heels and on toes; should be assessed for speed, stride length, step height, and arm swing option D= press or pull against resistance; grade from 0-5

62 years man -Double vision; condition: metastatic prostate cancer -Unable to adduct left eye, and stimulation of left cornea doesn't elicit a corneal reflex -Lesion involving? a) foramen lacerum b) foramen ovale c) foramen rotundum d) inferior orbital fissure e) optic canal f) superior orbital fissure

answer: superior orbital fissure -Absence of blink reflex on stimulation of left cornea, indicating lesion in afferent limb of reflex + eye adduction= superior orbital fissure -the sensory limb of the corneal reflex= mediated by the nasociliary branch of CN V1 (superior orbital fissure); motor component of corneal reflex= temporal branch of CN VII= bilateral eye blink -eye adduction= medial rectus= CN III (from midbrain and enters orbit via superior orbital fissure) -most patients with superior orbital fissure lesions= multiple deficits= 1) ophthalmoplegia (CN III, IV, VI) 2) ptosis (CN III, levator palpebrae), 3) fixed dilated pupil (sympathetics of CN III) 4) lacrimal hyposecretion and forehead numbness (V1) option A= carotid artery passes over it option B= CN V3 option C= CN V2; to pterygopalatine fossa to inferior orbital fissure to infraorbital foramen as infraorbital nerve option D= CN V2, infraorbital vessels, branches of sphenopalatine ganglion option E= optic nerve and ophthalmic artery

28 years woman -After vaginal delivery= frequent episodes of fecal incontinence, mild perineal pain (takes opioids for it) -Healing perineal laceration (no signs of infection) -Digital anorectal exam: decreased anal sphincter tone and loss of anal wink reflex -Cause of fecal incontinence? a)Cauda equina syndrome b)Lumbosacral plexopathy c)Opioid medication side effect d)Pudendal nerve injury e)Spinal epidural hematoma

answer: pudendal nerve injury -Pudendal nerve= motor (pelvic floor muscles and external urethral and anal sphincters) and sensory innervation to perineal region (external genitalia and skin around anus and perineum) -Originates from ventral rami of S2-S4 = passes between piriformis and coccygeus muscles as it exits pelvis through the greater sciatic foramen= then reenters the pelvis near the ischial spine through the lesser sciatic foramen before dividing into terminal branches -Pedundal nerve is vulnerable to stretch injury (due to curved course around ischial spine)= injury during prolonged labor second stage -External anal sphincter dysfunction= fecal incontinence and decreased anal sphincter tone -Perianal skin sensation dysfunction= loss of anal wink reflex option A= compression of spinal nerve roots below level of conus medullaris; unilateral or bilateral radicular pain, saddle anesthesia, lower extremity weakness and hyporeflexia. Bowel and bladder incontinence are late manifestations option B= direct compression during fetal descent; transient footdrop and numbness of lateral leg and dorsal foot option C= constipation; decreased intestinal motility due to activation of GI mu-opioid receptors option E= sudden-onset back pain or radicular pain with progression to complete or partial paralysis of lower extremities

54 years man -Dies= progressive neurological disorder -Structure with cystic degeneration (black arrow)? (picture in answer; no need to answer; just learn) a) globus pallidus b) putamen c) internal capsule d) caudate nucleus e) amygdala

answer: putamen -Many can cause cystic degeneration of putamen= Wilson's disease (most heavily tested) -Putamen: medial to the insula and lateral to globus pallidus and internal capsule option A= medial to putamen option C= white matter, separates globus pallidus and putamen from caudate nucleus anteriorly and from the thalamus posteriorly option D= near inferolateral walls of anterior horns of lateral ventricles; affected in Huntington's disease option E= group of neurons in medial temporal lobe; adjacent to hippocampus; part of limbic system; inferior to globus pallidus and putamen; mediates fear and some sympathetic stimulation

25 years man -Fractures his right tibia (motor vehicle accident) -Right leg fixed in cast (crutches) -2 weeks later= right upper extremity weakness and numbness -Right-handed (can't work) -Diminished strength on extension of right wrist with absent triceps reflex -Injury to which nerve? a) accessory b) axillary c) long thoracic d) median e) radial f) suprascapular

answer: radial -C5-T1; courses medial to surgical neck of humerus, inferior to the teres major muscle within the axilla before entering posterior arm to course between long head of triceps brachii and posterior humerus -Proximal radial nerve injury (within axilla); repetitive pressure/trauma caused by ill-fitting crutches (crutch palsy) or If an individual sleeps with the arm over a chair (Saturday night palsy) -Weakness of extensor muscles (arm below shoulder; wrist drop, absent triceps reflex); loss of sensation in posterior arm, forearm and dorsolateral hand -Distal radial nerve injury (midshaft humeral fracture) spares triceps brachii (triceps reflex intact) option A= sternocleidomastoid and trapezius option B= fracture of surgical neck of humerus and anterior dislocation of glenohumeral joint; paralysis of deltoid and teres minor (weakness on arm abduction); sensory loss over deltoid option C= paralysis of serratus anterior and winging of scapula; axillary lymph node dissection in patients undergoing radical mastectomy option D= proximal= supracondylar humerus fractures; sensory loss over palmar aspect of first 3 and half digits and impairment of thumb flexion/opposition, flexion of the second/third digits, and wrist flexion/abduction option E= innervates supraspinatus and infraspinatus; abduct and laterally rotate the arm, respectively

29 years woman -Worsening anxiety (anxious about everything) -Everything is normal -Fluoxetine is prescribed (patient improves)= inhibits reuptake of a neurotransmitter released by specific neurons? a)Caudate nucleus b)Locus coeruleus c)Nucleus basalis of Meynert d)Raphe nuclei e)Red nucleus f)Substantia nigra

answer: raphe nuclei -CNS serotonergic neurons= raphe nuclei -Located in midbrain, pons, medulla and axons from the cell bodies project throughout the CNS -Play a role in sleep-wake cycle, anxiety, mood, psychosis, sexuality, eating behavior, impulsivity -Primary pharmacotherapy for most depressive and anxiety disorders Antidepressants= 1) SSRIs (e.g., fluoxetine) 2) SNRIs 3) TCAs= work by prolonging the amount of time serotonin spends in synaptic cleft option A= striatum; damaged in Huntington disease; loss of cholinergic and GABA releasing neurons option B= release norepinephrine; located in dorsal pons option C= release acetylcholine; located in substantia innominate of basal forebrain; affected in Alzheimer option E= anterior midbrain; motor coordination of upper extremities option F= contains dopaminergic neurons; affected in Parkinson

16 years boy -Eye injury (baseball game) -Diplopia (worse when he looks up) -Mild periorbital ecchymosis, enophthalmos, limited vertical movement of right eye -Visual acuity is normal; CT= orbital floor fracture -Additional findings? a)Absent right corneal reflex b)Bruising over the mastoid process c)Impaired olfaction d)Inability to puff out right cheek e)Reduced sensation over right upper lip

answer: reduced sensatin over right upper lip -Fractures to orbital floor, composed of zygomatic bone and maxilla= from direct frontal trauma to orbit -The infraorbital nerve (continuation of maxillary nerve) runs along the orbital surface of maxilla in the infraorbital groove before traversing the infraorbital canal and exiting via the infraorbital foramen (below orbit) -Damage to it= paresthesia of upper cheek, upper lip, upper gingiva -in addition, displacement of orbital contents through floor= enophthalmos, entrapment of inferior rectus muscle (impair vertical gaze) option A= involves CN V1 (sensory) and VII (motor) and processing pons; impaired by trauma to the superior orbital fissure; because CN III, IV, V, VI pass through option B= hematomas over mastoid process (battle sign), along with periorbital ecchymosis and clear otorrhea, are signs of basilar skull fracture option C= nasal fractures or high force trauma to midface= loss of smell if the olfactory nerves are torn from the cribriform plate in ethmoid bone option D= CN VII; impaired in fractures of temporal bone after lateral head trauma or zygomatic orbit fractures

64 years man -Left lower extremity weakness (overnight) -No headache, double vision or back pain -History: hypertension, diabetes, MI -High BP and pulse (irregularly irregular) -Left lower extremity: 0/5 motor power; deep tendon reflexes are 3+, Babinski sign is present -Right lower extremity: normal -Abnormality in? a)Anterior spinal artery of the lumbosacral region b)Left posterior inferior cerebellar artery c)Lumbosacral spinal neuroforamina d)Right anterior cerebral artery e)Right middle cerebral artery

answer: right anterior cerebral artery -Patient with atrial fibrillation (irregularly irregular pulse, tachycardia) + isolated lower extremity motor defects= cardioembolic stroke in the right ACA -Occlusion of ACA= contralateral lower limb weakness with UMN signs (e.g., hyperreflexia, Babinski sign) -Most common cause of ACA stroke: cardiac embolization due to atrial fibrillation, can cause blood flow stasis resulting in thrombus formation in left atrial appendage which often travel and obstruct ACA or MCA option A= embolic occlusion= bilateral lower extremity weakness with hyporeflexia (ischemia of anterior horns and corticospinal tracts) and loss of pain and temperature sensation (spinothalamic tracts) below lesion option B= ischemia in lateral medulla (Wallenberg syndrome) and inferior cerebellar peduncle, resulting in dysphagia, hoarseness, and vestibulocerebellar symptoms (e.g., ataxia, dizziness, nystagmus) option C= can cause unilateral lower extremity weakness, would also cause leg pain, radicular back pain and hyporeflexia option E= predominantly upper extremities affected (sensory and motor); homonymous hemianopia, aphasia, hemineglect; larger MCA strokes affects both lower and upper extremities

55 years right-handed man -Severe, throbbing, right sided headache, double vision -History: poorly controlled hypertension and smoking -Position of right eye is down and out with ipsilateral ptosis, and right pupil is dilated and nonreactive to both light and accommodation -CT angiography: aneurysm arising from? a) junction of anterior communicating artery and anterior cerebral artery b) right middle cerebral artery c) left middle cerebral artery d) right junction of posterior communicating artery and internal carotid artery e) left junction of posterior communicating artery and internal carotid artery f) bifurcation of posterior cerebral artery g) right superior cerebellar artery h) left superior cerebellar artery

answer: right junction of posterior communicating artery and internal carotid artery -Right oculomotor nerve palsy secondary to a compressive aneurysm arising from junction of right posterior communicating artery with internal carotid artery -85% of saccular aneurysms affect anterior circulation (e.g., anterior communicating, posterior communicating, middle cerebral artery) -Chronic smoking and poorly controlled hypertension are risk factors -CN III exits midbrain between posterior cerebral and superior cerebellar arteries and courses along the Pcomm -CN III carries visceral efferent fibers (for pupillary light and near-reflex pathways) and general somatic efferent fibers within its interior (innervating superior rectus, inferior rectus, medial rectus, inferior oblique, and levator palpebrae superioris muscles) -Aneurysmal compression= mydriasis with diplopia, ptosis and down and out deviation in the ipsilateral eye option A= junction of Acomm and ACA; most common location of saccular aneurysms; compress optic chiasm= bitemporal hemianopia option E= Pcomm and internal carotid junction; left side= affect left eye

66 years right handed man -Left-sided numbness and weakness (left arm more than left leg) -History: hypertension, smoker (40 years) -High BP; impaired touch discrimination in left arm with left facial weakness sparing the forehead -2/5 muscle strength in left upper limb; and 4/5 muscle strength in left lower limb -Artery occluded? a) right anterior cerebral artery b) right middle cerebral artery c) right posterior communicating artery d) right posterior cerebral artery e) basilar artery f) anterior communicating artery g) left anterior cerebral artery h) left middle cerebral artery i) left posterior cerebral artery j) anterior inferior cerebellar artery

answer: right middle cerebral artery -right MCA occlusion= contralateral hemiparesis and hemisensory loss involving face and upper limb (little bit in lower limb) -Other features: contralateral hemineglect (nondominant parietal lobe); contralateral homonymous quadrantanopia (damage to optic radiations in subcortical temporoparietal lobe) -If it affects left MCA= speech areas damaged (expressive or receptive aphasia); right sided motor and sensory deficits)= choice H option A and G= ACA occlusion= contralateral motor and sensory deficits, lower limbs more than upper limbs; bilateral ACA occlusion= abulia, primitive reflexes (e.g., Moro, grasp) and urinary incontinence (due to damage to prefrontal cortex option C and F= saccular aneurysms affect anterior and posterior communicating arteries; Acomm= compress optic chiasm= bitemporal hemianopia; Pcomm= compress oculomotor nerve= ipsilateral mydriasis, ptosis and down and out gaze option D and I= PCA= contralateral homonymous hemianopia with macula sparing option E= basilar artery= damage to pons= quadriplegia, bulbar dysfunction (Facial weakness, dysarthria) and oculomotor deficits (horizontal gaze palsy) option J= a)anterior inferior cerebellar artery= lateral pontine syndrome= ipsilateral loss of pain or temperature in face (trigeminal nucleus), ipsilateral facial weakness (facial nucleus), ipsilateral hearing impairment (cochlear nucleus), contralateral loss of pain and temperature in trunk and extremities (lateral spinothalamic tract); cerebellar dysfunction (ataxia, dysmetria)

32 years man -Bowel dysfunction; motor vehicle accident (6 months ago) -Severe constipation with occasional fecal incontinence -Large amount of stool in distal colon due to impaired motor function -Anal sphincter tone is decreased -Injury to? a) cervical spinal cord b) pudendal nerve c) sacral spinal cord d) thoracic spinal cord e) vagus nerve nucleus

answer: sacral spinal cord -Both constipation and fecal incontinence with decreased tone of anal sphincter after traumatic injury -Deficiency of parasympathetic input or excess sympathetic input= constipation (impaired peristalsis) -Above splenic flexure= peristalsis controlled by parasympathetic innervation (vagus nerve) -Beyond splenic flexure= parasympathetic innervation from S2-S4 nerve roots -External sphincter= under voluntary control (pudendal nerve (S2-S4)) -Internal sphincter= sympathetic innervation normally maintains it in a state of tonic contraction; pressure in rectal vault (presence of stools)= stimulates parasympathetic input from pelvic splanchnic nerves to relax sphincter -Patient likely has impairment of both parasympathetic and voluntary motor output from S2-S4 nerve roots because of sacral spinal cord damage -Impaired peristalsis beyond splenic flexure (i.e., constipation; PSNS) and flaccid weakness of external anal sphincter (fecal incontinence; voluntary motor) option B= involved in external anal sphincter innervation; dysfunction= fecal incontinence

68 years woman -Lower extremity weakening; right femoral neck fracture and underwent total hip arthroplasty -History: hypertension, Type 2 diabetes, prior transient ischemic attack and osteoarthritis -Weakness in dorsiflexion, plantar flexion, and knee flexion -Hip flexion and knee extension are normal -Sensation to light touch= decreased on dorsum of right foot and posterolateral aspect of right calf -Ankle reflex absent on right -Cause? a)Diabetic neuropathy b)L5 radiculopathy c)Lacunar stroke d)Peroneal nerve compression e)Sciatic nerve injury

answer: sciatic nerve injury -Underwent total hip replacement following right femoral neck fracture and developed lower extremity deficits across sciatic nerve (L4-S3) and its main branches -Sciatic nerve: weakness in knee flexion -Common peroneal nerve: weakness in dorsiflexion, numbness on dorsal foot and posterolateral calf -Tibial nerve: weakness in plantar flexion and inversion, absent ankle reflex -Sciatic injury= especially in patients with femoral head dislocation, hip fracture, and/or arthroplasty due to its proximity to hip joint and femur option A= peripheral sensory loss (e.g., stocking-glove numbness); typically, bilateral, lower and upper extremities, and develop chronically with loss of distal sensory axons first, followed by motor deficits in advanced cases option B= caused by disc herniation or degenerative vertebral changes; unilateral lower extremity motor and sensory loss; patient have significant radicular back pain with normal ankle reflex and lateral foot sensation (both derived from S1 nerve root option C= a)due to chronic hypertension; unilateral sensorimotor deficits without cortical signs; result in complete hemiparesis/hemisensory loss (face, arm, leg) option D= against neck of fibula= in setting of prolonged immobilization and causes weakness in foot dorsiflexion, eversion and toe extension; and sensation from lateral leg and dorsum of foot

54 years woman -Easy fatigability; no weight gain or cold intolerance and no history of thyroid problems -History: gastrectomy due to nonhealing gastric ulcer -Exam: shiny tongue and pale palmar creases -Gait unstable when eyes are closed, vibration sense is decreased over lower extremities -Mild symmetrical loss of strength affecting proximal lower extremity muscles -Low Hb and WBCs; spinal cord lesion? a) poliomyelitis b) multiple sclerosis c) tabes dorsalis d) subacute combined degeneration of the spinal cord e) syringomyelia

answer: subacute combined degeneration of the spinal cord -Subacute combined degeneration of the spinal cord= chronic vitamin B12 deficiency -Low vitamin B12= pernicious anemia, gastrectomy, ileal resections, infection by tapeworm Diphyllobothrium latum, strict vegan diet -Vit. B12 deficiency= megaloblastic anemia, pancytopenia; elevated levels of methylmalonic acid (myelin synthesis abnormalities) -Affects dorsal columns (loss of position and vibration sensation, sensory ataxia (Positive Romberg sign) and lateral corticospinal tracts (spastic paresis) and spinocerebellar tracts (ataxia) option A= poliomyelitis and Werdnig-Hoffmann disease= destruction of LMNs in anterior horn= flaccid paralysis, muscle atrophy, and fasciculations option B= multiple sclerosis; demyelination of white matter of CNS in random and asymmetric patterns option C= tabes dorsalis (tertiary neurosyphilis); degeneration of dorsal root ganglia and dorsal columns; impaired vibratory sensation and proprioception option E= syringomyelia; central cavitation of cervical spinal cord; damage to anterior white commissure of spinothalamic tract; bilateral loss of pain/temperature in upper extremities; expansion into ventral horns= flaccid paralysis of hand muscles

72 years man -Involuntary movements of his right arm -Watching television when his arm "threw the remote control across the room" -Exam: wild, large amplitude, flinging movements affecting the proximal muscles of right arm -Area of lesion? a)Caudate nucleus b)Internal capsule c)Lentiform nucleus d)Substantia nigra e)Subthalamic nucleus f)Ventral posterior thalamus

answer: subthalamic nucleus -Subthalamic nucleus (below thalamus, above substantia nigra, medial to internal capsule); part of basal ganglia -Damage= decrease excitation of globus pallidus internus= reducing inhibition of thalamus= result in contralateral hemiballismus (wild, involuntary, large amplitude, flinging movements of proximal limbs (arm and/or leg) -Most common cause: lacunar stroke (long-standing hypertension and diabetes mellitus) option A= Huntington= chorea, athetosis, dementia, behavioral abnormalities option B= lacunar infarcts; contralateral pure motor or combined sensorimotor deficits option C= globus pallidus and putamen; Wilson disease (hepatocellular degeneration); liver (e.g., hepatitis, cirrhosis), psychiatric (e.g., depression, personality changes) and neurologic (e.g., dysarthria, movement disorder) option D= Parkinson; bradykinesia, rigidity, and resting tremor option F= lacunar infarcts in VPL and VPM= complete contralateral sensory loss; can cause thalamic pain syndrome

63 years man -long-standing osteoarthritis (refractory to medication) -Surgery= right total hip arthroplasty -After= difficulty walking despite minimal walking -Exam: leans to right side when walking -Asked to stand on right leg; left hip tilts downwards -Nerve injured? a) femoral b) inferior gluteal c) obturator d) sciatic e) superior gluteal

answer: superior gluteal -Superior gluteal nerve injury= pelvic trauma, or iatrogenic damage (hip surgery or butt injections) -Superior gluteal= innervates gluteus medius, gluteus minimus and tensor fasciae latae -Function= stabilize pelvis and abduct thigh -Injury= 1) pelvis sags towards normal side when patient stands (positive Trendelenburg sign) 2) when walking, the patient will lean toward the affected (ipsilateral) side to compensate for hip drop (gluteus medius lurch) option A= thigh flexion at hip (iliacus and sartorius); extension of leg at knee (quadriceps femoris); sensory= anterior thigh and medial leg; injury= knee buckling and loss of patellar reflex option B= gluteus maximus muscle; extension and external rotation of the thigh at the hip; injury= difficulty rising from seated position and climbing stairs option C= impaired thigh adduction and medial thigh sensory loss option D= muscles in posterior compartment of thigh (i.e., hamstrings); divides into tibial and common fibular nerves (motor and sensory innervation to leg and foot)

35 years man -Progressive right-hand weakness; uses screwdriver often (work) -Preserved sensation in upper limbs -Weakness of extension of fingers and thumb in right hand (otherwise everything is normal) -Triceps reflexes are 2+ and bilaterally symmetric -Nerve affected injured at? a) axilla b) carpal tunnel c) coracobrachialis d) hook of hamate e) midshaft of humerus f) supinator muscle

answer: supinator muscle -Radial nerve enters forearm anterior to lateral epicondyle and divides into superficial and deep branches -Superficial branch= purely sensory innervation to radial half of dorsal hand -Deep branch innervates extensor muscles of forearm; after passing between superficial and deep parts of supinator muscle, deep branch continues as posterior interosseous nerve, innervates muscles involved in finger and thumb extension -Injury to deep branch at supinator muscle may occur due to repetitive pronation/supination of forearm (e.g., frequent screwdriver use), direct trauma or dislocation head of radius -Patients typically have: weakness of finger and thumb extension (e.g., finger droop) -Triceps brachii (responsible for elbow extension and triceps reflex) and extensor carpi radialis longus (responsible for wrist extension) and sensory branches are unaffected because they diverge proximal to supinator muscle option A= weakness of forearm, hand, and finger extensor muscles (e.g., wrist drop) with absent triceps reflex and sensory loss over posterior arm, forearm and dorsolateral hand option B= compression of ulnar nerve (e.g., pregnancy, hypothyroidism); pain/paresthesia of first 3 and half digits with thenar atrophy and thumb abduction/opposition weakness option C= deep to biceps brachii and innervated by musculocutaneous nerve; nerve injury can result in decreased strength of forearm flexion and sensory loss over lateral forearm option D= ulnar nerve passes between hook of hamate and pisiform bone in Guyon canal; weakness on finger abduction/adduction and clawing of 4th and 5th digits option E= similar to A, except doesn't affect elbow extension and triceps reflex

36 years woman -Tingling, numbness of right hand and fingers that progressed to dull, aching pain (intermittent then persistent) -Secretary (difficulty typing and holding phone due to hand weakness) -Exam: weakness of thumb opposition and atrophy of thenar eminence -During surgery= incision of which structure will improve symptoms? a)Fibrous flexor sheaths b)Flexor carpi ulnaris aponeurosis c)Palmar aponeurosis d)Synovial sheaths of flexor tendons e)Transverse carpal ligament

answer: transverse carpal ligament -Carpal tunnel syndrome (carpal tunnel= between carpal bones and transverse carpal ligament/flexor retinaculum) -Median nerve and 9 tendons pass through carpal tunnel to enter hand -Often CTS can be improved without surgery (e.g., wrist splint, glucocorticoid injection) -Severe impairment often treated with surgical decompression (incision through transverse carpal ligament) option A= thick tunnels on palmar surface of fingers; allows flexor digitorum profundus tendons to flex the fingers at the interphalangeal joints; swelling of tendon and/or fibrous sheath= painful catching or locking of affected finger during flexion option B= from olecranon to medial epicondyle and forms the roof of cubital tunnel; ulnar nerves runs through it option C= fibrosis of palmar aponeurosis= Dupuytren's contracture, progressive disorder characterized by nodular lesions in the palm with restricted finger motion option D= pass through carpal tunnel around flexor tendons; incision is useless in CTS

37 years man -Enlarged cervical lymph nodes, fever, fatigue, drenching night sweats -Undergoes excisional biopsy of enlarged nodes in the left posterior triangle of neck -2 weeks later= difficult with overhead activities such as combing hair, or placing dishes on overhead shelves -Exam: left shoulder droop with weakness of left arm abduction above 100 degrees -Other shoulder movements are normal; no sensory loss -Muscle paralyzed? a) deltoid b) latissimus dorsi c) levator scapulae d) serratus anterior e) trapezius

answer: trapezius -Spinal accessory nerve (CN XI)= pure motor nerve (sternocleidomastoid and trapezius) -Runs in posterior triangle of neck= vulnerable to iatrogenic injury (e.g., cervical lymph node dissection) -Trapezius elevates and rotates the scapula upward (e.g., during arm abduction) and stabilizes the shoulder --> weakness= drooping of the shoulder, impaired abduction of arm above 100 degrees, and lateral displacement of scapula -if injury involves the proximal portions of nerve= weakness of sternocleidomastoid option A= innervated by axillary nerve (C5-6); impaired sensation over lateral shoulder and weakness of abduction of arm from 30-100 degrees; iatrogenic injury during shoulder surgery (e.g., proximal humerus fracture repair) or intramuscular injection in deltoid option B= innervated by thoracodorsal nerve (damaged during axillary lymph node dissection)= most powerful adductor of arm and assists with extension and medial rotation option C= innervated by C3-C4 cervical nerves and dorsal scapular nerve (C5); acts to elevate scapula and rotate its medial border inferiorly, no effect on arm abduction option D= innervated by long thoracic nerve; protracts and rotates scapula upward, assisting with elevation of arm over head= causes winging of scapula (weakness); after lymph node dissection and would not be associated with shoulder droop

71 years man -Burning pain, then 2 days later rash on his right forehead -Vesicular rash on right forehead and right side of nose -Risk of blindness in right eye due to which cranial nerve? a) abducens b) facial c) oculomotor d) optic e) trigeminal f) trochlear

answer: trigeminal -Herpes zoster ophthalmicus, caused by reactivation of latent varicella-zoster virus (VZV) in trigeminal ganglion -Primary VZV infection (i.e., chickenpox), virus migrates via sensory nerves to cranial nerve and dorsal spinal ganglia, where it lies dormant for years -Reactivation of virus= occurs in a single ganglion, and causes painful, vesicular rash in a dermatomal distribution -Viral reactivation within the trigeminal ganglion (most often V1)= vesicular rash within V1 distribution (e.g., unilateral forehead, side of nose, periorbital area) -CN V1 also relays sensory innervation of cornea (sensory limb of corneal/blink reflex)= involvement= acute keratitis, which can lead to vision loss -Vesicular lesions on side and tip of nose (i.e., Hutchinson sign) have high correlation with subsequent eye involvement (nasociliary branch of CN V1; innervates eye and side/tip of nose) option B= affected in herpes zoster oticus (Ramsay Hunt syndrome)= ear pain, vesicles in external auditory canal or auricle, facial paralysis; CN VII innervates orbicularis oculi (so facial nerve paralysis are at risk of exposure keratitis= blindness option D= in rare cases; blindness due to acute retinal necrosis and/or neuritis of optic nerve

65 years man -Acute-onset slurred speech -Right-sided weakness (no trauma, headache, or loss of consciousness) -History: hypertension, T2D, smoker (20 years) -Exam: right-sided lower facial droop with sparing of forehead muscles -Motor strength 3/5 on right and 5/5 on left with Babinski response on right -Dysmetria and dysdiadochokinesia (right upper and lower extremities) -MRI= acute lacunar infarct in lateral-mid-pons at level of middle cerebellar peduncle -Which cranial nerve exits brainstem closest to level of lesion? a) facial b) hypoglossal c) oculomotor d) trigeminal e) trochlear

answer: trigeminal -Patient has acute lacunar ischemic stroke affecting left medial pons at level of middle cerebellar peduncle -Trigeminal nerve exits brainstem at lateral aspect of mid-pons at level of middle cerebellar peduncle (a key neuroanatomic landmark for locating the nerve) -Trigeminal sensory nuclei (principal sensory, spinal, mesencephalic) run from midbrain to upper cervical spine and receive signals for facial sensation via all 3 nerve branches -Motor nucleus located in lateral mid-pons and sends efferent signals to muscles of mastication (e.g., temporalis, masseter, pterygoids) via mandibular branch -Infarcts involving anterior pons= 1) corticospinal tract (contralateral hemiparesis, Babinski sign) 2) corticobulbar tract (contralateral lower facial palsy, dysarthria) 3) corticopontine fibers (contralateral dysmetria, dysdiadochokinesia) -Cerebellar deficits are contralateral to lesion as pontocerebellar fibers arising from pontine gray matter decussate and enter cerebellum through contralateral middle cerebellar peduncle option A= dorsolateral aspect of caudal pons; nerve exits at ventrolateral pontomedullary junction (below middle cerebellar peduncles); patient has sparing of forehead muscles option B= medially on the floor of the 4th ventricle at level of medulla, nerve exits the rostral end of preolivary sulcus option C= rostral midbrain at level of superior colliculus and red nucleus; nerve exits at the interpeduncular fossa option E= caudal midbrain; nerve exits dorsal midbrain just below the inferior colliculus (below red nucleus); only cranial nerve to decussate before reaching target

53 years man -Double vision; difficulty walking up stairs (duplicates of every step) -Trouble reading (newspaper, work-related documents) -Lesion in? a)Abducens nerve b)Medial longitudinal fasciculus c)Oculomotor nerve d)Optic nerve e)Trochlear nerve

answer: trochlear nerve -Trochlear nerve= superior oblique muscle (intorsion and depress while adducted) -Present: vertical diplopia (noticeable when eye looks down and towards nose- reading, walking down the stairs) -Exam: impairment of downgaze with the eye in adducted position; vertically upward deviation (hypertropia) -Symptoms improve when chin is tucked, and head is tilted away from affected eye (compensates hypertropia and extortion) option B= internuclear ophthalmoplegia

56 years man -Weakness and difficulty speaking -Headache while smoking crack previous night -Morning= unable to move or speak -Exam: awake with normal pupillary responses -Vertical eye movements (normal), horizontal eye movements= impaired -Loss of motor strength in upper and lower extremities bilaterally -Ischemic infarction (cocaine-induced vasospasm) of which region? a) angular gyrus b) cingulate gyrus c) ventral midbrain d) ventral pons e) medial medulla f) lateral medulla

answer: ventral pons -Quadriplegia and speechlessness with preserved consciousness and eye movements= locked-in syndrome due to ischemic injury to bilateral ventral pons -Locked in syndrome: 1) quadriplegia and loss of speech (corticospinal and corticobulbar tracts) 2) absent horizontal eye movements (vertical eye movements and eyelid elevation= rostral midbrain) 3) preserved consciousness (midbrain reticular formation is spared= choice C) 4) preserved sensation option A= in dominant parietal lobe= Gerstmann syndrome (agraphia, acalculia, finger agnosia, left-right disorientation option B= significant behavioral symptoms (e.g., abulia) option E= contralateral hemiparesis (corticospinal tract), contralateral hemisensory loss (dorsal column/medial lemniscal pathway) and ipsilateral tongue paralysis (CN XII) option F= Wallenberg syndrome; vertigo/nystagmus, ipsilateral cerebellar signs (e.g., ataxia, dysmetria), loss of pain/temperature sensation in ipsilateral face and contralateral body, bulbar weakness (e.g., dysphagia) and ipsilateral Horner syndrome (miosis, ptosis, anhidrosis)

question 1 of 2 62 years woman -Difficulty walking (3 hours ago); can't feel right side of body -History: hypertension, diabetes mellitus, smoker (30 years) -Father: MI at age 60 -Exam: loss of touch, temperature, vibratory sensation (right upper and lower extremities); sensation diminished over right side of face; muscle strength: 5/5 -Stroke to? a) base of pons b) caudate nucleus c) frontal cortex d) posterior limb of the internal capsule e) ventral posterior thalamus

answer: ventral posterior thalamus -Right-sided pure hemisensory loss= thalamic stroke -The thalamic ventral posterior lateral nucleus (receives input from spinothalamic tract and dorsal columns) and ventral posterior medial nucleus (receives input from the trigeminal pathway)= send somatosensory projections to cortex via thalamocortical fibers -Damage to these nuclei (ischemia, hemorrhage, etc.)= complete contralateral sensory loss (e.g., touch, pain/temp., vibration/proprioception); some proprioceptive defects= unsteady gait option A= contralateral weakness and ataxia due to involvement of descending motor tracts (e.g., corticospinal, corticobulbar) and pontocerebellar fibers option B= a)behavioral abnormalities (e.g., agitation, psychosis, abulia), speech/language disturbances, and movement disorders (e.g., choreoathetosis) option C= social disinhibition, deficits in attention, disturbed executive function (e.g., planning and decision making) option D= anterior 2/3 of posterior limb= motor fibers (e.g., corticospinal tract) and posterior 1/3 contains sensory fibers (e.g., thalamocortical tract); although small lesions of posterior limb may cause contralateral sensory deficits, most lesions= pure motor stroke or combined sensorimotor deficits

5 years boy -Persistent food seeking behavior (never satisfied); weight gain -Headache, nausea (morning); high BMI -Lesion of which hypothalamic nucleus? a) anterior b) lateral c) posterior d) suprachiasmatic e) supraoptic f) ventromedial

answer: ventromedial -Hypothalamus (diencephalon): important in homeostasis -Ventromedial nucleus is vital in monitoring blood glucose and mediates satiety= lesions (hyperphagia and obesity)= Very Massive -Lateral= signals hunger= lesions (anorexia)= Lean -Anterior= cooling via inhibition of adrenergic input, causing vasodilation and stimulation of cholinergic pathways (e.g., sweating)= lesion (hyperthermia)= A/C -Posterior= heat conservation and heat production via vasoconstriction and shivering= lesion (hypothermia)= purns - Suprachiasmatic= above optic chiasm, receives visual input from retina= regulates circadian rhythms by relaying light info to other hypothalamic nuclei and pineal gland= suprakiasmatic= sarkadian rhythm -Supraoptic and paraventricular= project to posterior pituitary= release ADH (fluid balance) and oxytocin (uterine contractions and milk letdown)

30 years man -Headache and neck pain (following MVA) -Neuroimaging= right transverse foramina fracture at level of C2 -While in hospital= dizziness, right facial numbness and hoarseness -Neurologic exam: partial ptosis, miosis of right eye, nystagmus; hearing is intact bilaterally -Reduced sensation to pain and temperature on right side of face and left side of body -Ataxic and displays past-pointing with his right hand on finger-to-nose testing -Dissection of? a) basilar b) common carotid c) internal carotid d) middle cerebral e) posterior cerebral f) vertebral

answer: vertebral -findings= ischemia of right posterior inferior cerebellar artery (arises from vertebral artery)= lateral medullary (Wallenberg) syndrome -Features: vertigo/nystagmus, ipsilateral cerebellar signs (e.g., ataxia), loss of pain/temperature sensation in ipsilateral face and contralateral body, bulbar weakness (e.g., dysphagia, dysphonia); patients may also develop ipsilateral Horner syndrome due to damage of sympathetic fibers innervating head -Traumatic vascular dissection (common ischemic stroke in young)= affects extracranial vessels mostly (e.g., carotid and vertebral arteries) -Vertebral arteries course through the transverse foramina of neck before entering skull at foramen magnum= easily injured in cervical spinal trauma -Progressive dissection can ascend intracranially= affecting PICA option A= i.e., lateral pontine syndrome) due to basilar artery dissection could cause ataxia, nystagmus, vertigo, loss of facial sensation); it is very rare; also have hearing loss and facial paralysis option B and C= dissection can occur with cervical spinal injuries; but profound neurologic deficits (e.g., contralateral homonymous hemianopsia, hemiparesis, hemisensory loss) option D= dissection rare; typically occluded by atherothrombosis or cardioembolism; contralateral hemiparesis , and hemisensory loss (face and arm> Leg), conjugate gaze deviation, contralateral homonymous hemianopsia, aphasia (if dominant) option E= dissection rare; occluded by atherothrombosis or cardioembolization; contralateral homonymous hemianopsia with macular sparing

34 years man -Persistent buzzing in right ear for past 6 months -Decreased hearing on right side -Otoscopic exam: normal appearing external auditory canals bilaterally with pinkish-grey tympanic membranes -MRI= intracranial tumor -Lesion arisen from? a) trigeminal nerve b) abducens nerve c) facial nerve d) vestibulocochlear nerve e) glossopharyngeal nerve f) vagus nerve g) spinal acessory nerve

answer: vestibulocochlear nerve -Acoustic neuroma (schwann cell-derived tumor; typically arises from the vestibular potion of the vestibulocochlear nerve) -CN VIII exits brainstem from pontomedullary junction, lateral to facial nerve, enters cerebellopontine angle -Acoustic neuroma= ipsilateral sensorineural hearing loss and tinnitus due to impairment of cochlear potion of CN VIII -Damage to vestibular potion= vertigo, disequilibrium, nystagmus -Bilateral acoustic neuroma= NF type 2

64 years man -Low back pain and buttocks (worsens with standing or walking and improves when lying down) -Occasional sharp, shooting pain that radiates down his right leg -Takes too many breaks at work -Spine imaging= degenerative joint changes and a large osteophyte compressing the spinal nerve root exiting through the right neural foramen between the L5 and S1 vertebrae -Findings? a)Decreased knee reflex b)Decreased rectal tone c)Sensory loss in inguinal region d)Weakness of foot dorsiflexion e)Weakness of hip flexion

answer: weakness of foot dorsiflexion -Lumbosacral radiculopathy= compression of L5 nerve root (at L5-S1 neural foramen); at lumbosacral spine= nerves exit below corresponding vertebral body -Mechanisms: 1) spinal spondylosis: degenerative changes and osteophyte formation can narrow neural foramina= leads to compression of nerve as it passes b/w neural foramina 2) vertebral disc herniation: tear in intervertebral disc annulus, leads to herniation of nucleus pulposus (if it occurs laterally= compress nerve exiting neural foramina (if it occurs centrally= it can protrude into spinal canal and compress other nerves in the cauda equina that exit one or more levels below the area of herniation) -L5 radiculopathy= 1) sensory loss and back pain that radiates down the leg (e.g., buttocks, lateral thigh and calf, dorsal foot) 2) tibialis anterior (foot dorsiflexion and inversion), peroneus (foot eversion), and extensor hallucis longus (Great toe extension) option A= mediated by quadriceps muscle= L2-L4 option B= damage to pedundal nerve= S2-S4 option C= L1 that travels in the ilioinguinal and genitofemoral nerves option E= iliopsoas= L2-L4

53 years man -Pins and needles sensation in his right hand (worse at night) -No neck pain, or history of trauma to right arm -Smoker (20 years), accountant (30 years) -Loss of sensation over the right fifth digit -Weakness of which of following movements? a) elbow extension b) shoulder abduction c) wrist abduction d) wrist adduction e) wrist extension

answer: wrist adduction -Ulnar nerve: medial cord of brachial plexus= with brachial artery in upper arm= enters forearm after passing posterior to medial epicondyle of humerus (cubital tunnel)= innervates flexor carpi ulnaris (wrist flexion and adduction) and medial portion of flexor digitorum profundus= enters Guyon canal (between the hook of hamate and pisiform)= in the hand, superficial branch of the ulnar nerve provides cutaneous innervation to fifth digit and medial half of 4th digit and the hypothenar eminence -Deep branch supplies most the intrinsic muscles of the hand -Most commonly injured at the elbow due to trauma (e.g., medial epicondyle fracture) or compression (e.g., resting on a hard surface while using a computer) -Nerve can also be injured in the Guyon canal, resulting in more pronounced clawing of the 4th and 5th digits (ulnar claw) due to sparing of the flexor digitorum profundus option A and E= radial nerve injury (e.g., C6/C7 disc herniation, mid-shaft humerus fracture)= weakness in elbow, wrist extension; sensory loss over posterior arm, dorsolateral hand and dorsal thumb option B= axillary nerve injury (e.g., shoulder dislocation, humeral fracture); loss of sensation over lateral upper arm and weakness in shoulder abduction (deltoid weakness option C= proximal median nerve injury (e.g., trauma)= weakness of wrist flexion and abduction (radial deviation), flexion of 2nd/3rd digits; flexion/opposition of thumb along with palmar sensory loss first 3 and half digits


Ensembles d'études connexes

Ch 6 Quiz, Ch 7 Quiz, Ch 8 Quiz, Ch 10 Quiz, Ch 11 Quiz

View Set